geometria plana

108
curso de post-grado para profesores especialidad en matem ´ atica GEOMETR ´ IA EUCLIDEANA Equipo de Dise˜ no: Nahomy Jhopselyn Hern´ andez Cruz Gabriel Alexander Chicas Reyes Eduardo Arnoldo Aguilar Ca˜ nas ector Enmanuel Alberti Arroyo Ernesto Am´ erico Hidalgo Castellanos Juan Agust´ ın Cuadra Claudia Patricia Corcio L´ opez de Beltr´ an Carlos Mauricio Canjura Linares Oscar Armando Hern´ andez Morales Aar´ on Ernesto Ram´ ırez Flores 15 de marzo de 2010 ´ Indice 1. ´ Angulos entre paralelas. 2 2. Tri´ angulos: Teoremas Fundamentales. 7 3. Congruencia de Tri´ angulos. 16 4. Cuadril´ ateros: Clasificaci´on y Propiedades. 24 5. ´ Angulos en la Circunferencia. 30 6. Teorema de Thales y su rec´ ıproco. Semejanza de Tri´ angulos. 46 7. Puntos y Rectas Notables del Tri´ angulo. 58 8. Soluci´ on a Problemas Selectos. 72 1

Upload: cesar-orellana-rivas

Post on 30-Oct-2014

133 views

Category:

Documents


3 download

DESCRIPTION

todo lo básico de geometria Euclidiana

TRANSCRIPT

Page 1: Geometria Plana

curso de post-grado para profesoresespecialidad en matematica

GEOMETRIA EUCLIDEANA

Equipo de Diseno:Nahomy Jhopselyn Hernandez Cruz

Gabriel Alexander Chicas ReyesEduardo Arnoldo Aguilar CanasHector Enmanuel Alberti Arroyo

Ernesto Americo Hidalgo CastellanosJuan Agustın Cuadra

Claudia Patricia Corcio Lopez de BeltranCarlos Mauricio Canjura Linares

Oscar Armando Hernandez MoralesAaron Ernesto Ramırez Flores

15 de marzo de 2010

Indice

1. Angulos entre paralelas. 2

2. Triangulos: Teoremas Fundamentales. 7

3. Congruencia de Triangulos. 16

4. Cuadrilateros: Clasificacion y Propiedades. 24

5. Angulos en la Circunferencia. 30

6. Teorema de Thales y su recıproco. Semejanza de Triangulos. 46

7. Puntos y Rectas Notables del Triangulo. 58

8. Solucion a Problemas Selectos. 72

1

Page 2: Geometria Plana

1. Angulos entre paralelas.

ANGULOS

Definimos como angulo a la figura geometrica formada por dos rayos (o semirrectas) distintasque tienen el mismo origen. Ese origen se llama vertice del angulo. Al angulo de vertice O yrayos OA y OB se le denota ∠AOB.

Dos angulos ∠AOB y ∠BOC son adyacentes si y solo si tienen un lado comun OB y los ladosno comunes OA y OC estan en semiplanos distintos, determinados por el lado comun.

Bisectriz de un angulo es la semirrecta que lo “divide” en dos angulos adyacentes iguales.

Dos angulos son:

Congruentes o Iguales : si tienen igual medida.

Suplementarios : si su suma es 180°.

Complementarios : si su suma es 90°.

Por otra parte, dos rectas en el plano pueden ser secantes o paralelas,1 dependiendo si se cortano no; ademas, si las rectas son secantes, el punto de corte es unico, y definen cuatro angulos,que se agrupan por parejas en angulos opuestos por el vertice (las parejas de angulos tales queuno esta formado por la prolongacion de los lados del otro).

Los angulos opuestos por el vertice son iguales (Justifique), por lo que dos rectas secantes for-man cuatro angulos que definen dos parejas de angulos iguales, y si tomamos un miembro decada pareja, se tienen dos angulos suplementarios. En particular, si las rectas son secantes yforman cuatro angulos iguales, seran llamadas rectas perpendiculares,2 y los angulos ası gene-rados son llamados angulos rectos. Y como es muy conocido, un angulo agudo es aquel cuyamedida es menor a la de un angulo recto, y un angulo obtuso es aquel cuya medida es mayorque un angulo recto; en particular, un angulo obtuso sera llamado angulo llano si su medida esel doble que la de un angulo recto.

ANGULOS ENTRE PARALELAS

Al intersecar un par de rectas paralelas por una recta llamada transversal o secante, se formanlos siguientes tipos de angulo:

Angulos Correspondientes : Son dos angulos no adyacentes situados en el mismo lado dela secante, uno en el interior y otro en el exterior de las paralelas.

Angulos Alternos Internos : Son dos angulos no adyacentes situados en el interior de lasparalelas, y en distintos lado de la secante.

1Si la recta AB es paralela a la recta CD, se denota AB ‖ CD.2Si la recta AB es perpendicular a la recta CD, se denota AB ⊥ CD.

2

Page 3: Geometria Plana

Angulos Alternos Externos : Son dos angulos no adyacentes situados en el exterior de lasparalelas, y en distintos lado de la secante.

Angulos Conjugados : Son los angulos no adyacentes situados uno en el interior y el otroen el exterior de las rectas paralelas y del mismo lado de la secante.

Las propiedades fundamentales de los angulos entre paralelas son:

1. Los angulos correspondientes son iguales entre sı.

2. Los angulos alternos internos son iguales entre sı.

3. Los angulos alternos externos son iguales entre sı.

4. Los angulos conjugados son suplementarios.

Figura 1: Angulos entre las rectas paralelas L1 y L2.

Ejercicios

1. Tres angulos adyacentes forman un semiplano y tienen sus medidas proporcionales a losnumeros 5, 7 y 8. Hallar la medida del menor angulo.

2. Demostrar que las bisectrices de dos angulos suplementarios son perpendiculares.

3. En la figura adjunta, L1 ‖ L2 yL3 ‖ L4. Calcular x.

3

Page 4: Geometria Plana

4. Con ayuda de la figura 2, demuestre que: Si L1 ‖ L2 entonces γ = α + β.

Figura 2

5. En la figura 3, AB ‖ FG. Hallar el angulo x si el ∠AMF = 90° y el ∠MAB = 110°.

Figura 3

6. Calcular el ∠OPQ, si OP es bisectriz del angulo O, L1 ‖ L2 y PQ ⊥ L1. Ver figura 4.

Figura 4

4

Page 5: Geometria Plana

7. En la figura 5, L1 ‖ L2 y L3 ‖ L4, calcular α.

Figura 5

8. En la figura 6, calcular x, si L1 ‖ L2.

Figura 6

9. Calcular la medida θ del grafico anexo, si lasrectas L1 y L2 son paralelas.

5

Page 6: Geometria Plana

10. En la figura 7, L1 ‖ L2 y L3 ‖ L4. Hallar el valor del angulo θ.

Figura 7

11. Sea ∠AOB = 24°, en la region exterior a dicho angulo se traza el rayo OC. Hallar lamedida del angulo formado por las bisectrices de los angulos AOC y BOC.

12. Del grafico 8, calcular y, cuando x tome su maximo valor entero.

Figura 8

6

Page 7: Geometria Plana

2. Triangulos: Teoremas Fundamentales.

TEOREMAS FUNDAMENTALES EN TODO TRIANGULO.

Diremos que tres puntos que pertenecen a una misma recta son puntos colineales ; de maneraanaloga, si tres rectas pasan por un mismo punto, seran llamadas rectas concurrentes. Si toma-mos “al azar” tres puntos en el plano, en muy raras ocasiones estos puntos estaran alineados,3

y diremos entonces que son los vertices de un triangulo; analogamente sucede con las rectas,tres rectas por lo general no concurren, y la figura geometrica que estas definen es tambien untriangulo.4 Una definicion completa para nuestros intereses es la siguiente:

Definicion de Triangulo. Si A, B y C son tres puntos cualesquiera no colineales (Ver figura9), entonces la reunion se los segmentos AB, BC y AC se llama triangulo ABC y se denotapor 4ABC. Los puntos A, B y C se llaman vertices y los segmentos AB, BC y AC se llamanlados. Simbolicamente: 4ABC = AB∪BC ∪AC. Todo triangulo ABC determina tres angulosinternos o interiores : ∠ABC, ∠ACB y ∠BAC, y se llamara angulo externo o exterior, alangulo determinado por un lado y la prolongacion del lado adyacente, en la figura 9, α, β y θson angulos exteriores.

Figura 9: Elementos del Triangulo

Dado el 4ABC, se tiene que AB + BC + CA = p = 2s, donde p es llamado el perımetro y sel semiperımetro del triangulo. Para abreviar, suele asociarse a cada vertice un lado opuesto,y viceversa, por ejemplo, el lado opuesto de A es BC, y es frecuente que se denote por a;analogamente b = CA, c = AB.

Teorema 1: En todo triangulo, la medida de un angulo exterior es igual a la suma de lasmedidas de dos angulos interiores del triangulo no adyacentes a el.

La demostracion de este teorema se basa en las relaciones de angulos entre paralelas; se dejaal lector que haga la demostracion (Sugerencia: por un vertice, trace una recta paralela al lado

3En teorıa de probabilidades, ¡la probabilidad que esto ocurra es cero!4El termino mas riguroso para esta figura es trilatero. En este caso, habrıa que hacer una consideracion: si

hay un par de rectas paralelas, el trilatero definido ya no es “normal” segun nuestro sentido comun, sin embargo,¡sigue siendo un trilatero!

7

Page 8: Geometria Plana

opuesto)

Corolario: En todo triangulo, la suma de las medidas de sus tres angulos internos es igual a180°.

Teorema 2: Desigualdad Triangular. En todo triangulo, la longitud de uno de sus ladosesta comprendido entre la suma y la diferencia de los otros dos.

Sin ser muy rigurosos, suponga que dado el segmento AB se traza con centro en A una cir-cunferencia de radio r1, y con centro en B una circunferencia de radio r2; si AB < r1 + r2,las circunferencias se cortaran en dos puntos, y cualquiera de ellos puede ser el vertice C,ası AB < BC + CA; en cambio, si AB = r1 + r2 o peor aun, si AB > r1 + r2, la construcciondel 4ABC no es posible.

La Desigualdad Triangular es un resultado fundamental, a partir de esta y de su modelo dedemostracion se generan los Criterios de Congruencia de Triangulos ; a groso modo, si dadasciertas condiciones, la construccion de una figura geometrica (un triangulo en particular) quedadeterminada de manera unica, entonces dos figuras que reunen las mismas condiciones seranllamadas figuras congruentes.

Ası, si se tienen tres segmentos (cuyas longitudes cumplen la desigualdad triangular), dejandouno fijo y construyendo las circunferencias con centros en los extremos de este segmento yradios las longitudes de los otros segmentos, por construccion, solo sera posible obtener dostriangulos (uno con cada punto de interseccion de las circunferencias), que son basicamente elmismo pero la orientacion de los angulos es contraria; ası, si se sabe que dos triangulos cumplentener lados respectivamente iguales, por construccion, deben de ser iguales. Este es el conocidocriterio LLL de congruencia de triangulos; mas adelante se detallaran el resto de criterios, peroa partir de este probaremos el siguiente resultado:

Teorema 3: En todo triangulo, se cumple que a lados iguales se oponen angulos iguales, yviceversa.

Suponga que 4ABC es tal que AB = AC, entonces, por criterio LLL, 4ABC es congruenteal 4ACB (en ese orden, porque AB = AC, BC = CB y CA = BA), entonces, los angulosque se oponen a los angulos iguales son iguales. Para el recıproco necesitamos otro criterio decongruencia, por lo que la demostracion se dejara incompleta; retome esto en la seccion decongruencia de triangulos.

Teorema 4: En todo triangulo se cumple que a mayor lado se opone mayor angulo y viceversa.

Este teorema se deja como ejercicio para el lector (Sugerencia: utilice el teorema anterior, tomeel lado mayor y defina un punto adecuado que genere un triangulo con dos lados iguales.)

CLASIFICACION DE TRIANGULOS.

1. Con relacion a sus lados:

8

Page 9: Geometria Plana

a) Escaleno: si sus tres lados no son congruentes.

b) Isosceles : si por lo menos dos de sus lados son congruentes.

c) Equilatero: si sus tres lados son congruentes (note un triangulo equilatero es tambienisosceles, y que los tres angulos internos son iguales entre sı e iguales a 60)

2. Con relacion a sus angulos internos:

a) Acutangulo: si su angulo mayor es agudo (note que entonces los tres angulos sonagudos)

b) Rectangulo: si su angulo mayor es angulo recto (note que el angulo en cuestion esunico y que los otros dos angulos son agudos; ası, en un triangulo rectangulo, lahipotenusa es mayor a los catetos)

c) Obtusangulo, si el angulo mayor es angulo obtuso (note que el angulo en cuestiones unico y que los otros son agudos; ası, en un triangulo obtusangulo, el lado que seopone al angulo obtuso es el lado mayor)

LINEAS NOTABLES EN UN TRIANGULO.

1. Altura: Se llama altura de un triangulo al segmento que parte de uno de sus vertices yllega en forma perpendicular al lado opuesto o a su prolongacion.

2. Mediana: Se llama Mediana al segmento que une un vertice con el punto medio del ladoopuesto.

3. Mediatriz: Se denomina mediatriz de un lado de un triangulo es la recta perpendicular adicho lado en su punto medio.

4. Una Bisectriz: La bisectriz es la recta que “divide” en dos angulos iguales a un angulodado; en particular, es bisectriz interna si es la bisectriz de un angulo interno de untriangulo, y bisectriz externa si es la bisectriz de un angulo externo de un triangulo.

DISTANCIA DE UN PUNTO A UNA RECTA.

En la figura 10, sea P un punto exterior a una recta L, la longitud de la perpendicular PM ala recta L es la distancia del punto P a dicha recta. Esta perpendicular tiene la propiedad deser unica y su longitud es la distancia mınima del punto a la recta (Pruebelo utilizano el hechoque la hipotenusa es mayor que los catetos).Los segmentos PA y PB no son perpendiculares a L y se llaman oblicuas.

TEOREMA DE PITAGORAS.

Abordamos el estudio de las Relaciones Metricas, del cual solo realizaremos el analisis del fa-moso Teorema de Pitagoras, cuyo enunciado es el siguiente:

Teorema: Pitagoras. En un triangulo rectangulo, el cuadrado de la hipotenusa es igual a lasuma de los cuadrados de los catetos.

9

Page 10: Geometria Plana

Figura 10

Una demostracion de este teorema es debida a Thabit ibn Qurra (836-901), la cual consiste endiseccionar la figura que se forma al construir dos cuadrados de lados respectivamente igualesa los catetos de un triangulo rectangulo, como se muestra en el grafico 11.

Figura 11

Recıproco del teorema de Pitagoras: Si en un triangulo el cuadrado de un lado es igual ala suma de los cuadrados de los otros dos lados, el triangulo es rectangulo.5

Ejercicios.

1. En la figura adjunta ambos triangulos sonequilateros. Encuentre el valor de ϕ.

5Ver demostracion en la seccion de congruencia de triangulos.

10

Page 11: Geometria Plana

2. En la figura 12, calcular el ∠x si el ∠AOB = 100° y L1 ‖ L2.

Figura 12

3. (*) En la figura 13, ABDE es un cuadrado y BCD es un triangulo isosceles con BD =DC. Si ∠ABC = 160°, determinar la medida de ∠AEC.

Figura 13

4. (*) (XV Competencia de Clubes Cabri Primera Ronda) En la figuraadjunta, ABCD es un rectangulo tal que AB = 2BC. M es el puntomedio de AB y los triangulos AME y MBF son equilateros. Si Pes la interseccion de las rectas DE y CF , encuentre los angulos del4CDP .

5. Si AB y FG son rectas paralelas, el ∠ABC = ∠CDE = θ, el ∠DEF = θ2

y el ∠GFH =150°. Calcule θ. Figura 14

6. Probar que una bisectriz exterior de un triangulo es paralela al lado opuesto si y solo siel triangulo es isosceles.

11

Page 12: Geometria Plana

Figura 14

7. (*) Hallar la suma de los angulos α + ε+ θ + φ en la figura 15.

Figura 15

8. Determine el valor de la suma ∠A+ ∠B + ∠I + ∠H + ∠F + ∠G. Figura 16.

Figura 16

12

Page 13: Geometria Plana

9. En el 4ABC el ∠BAC = 36° y AC = AB. Probar que la bisectriz interior BD, D enAC, es congruente con el lado BC.

10. Sea ABC un triangulo rectangulo en B con AB = BC, se construye exteriormente eltriangulo equilatero BCD. Encuentre el angulo ∠DAB.

11. En el 4ABC, AB = AC y D un punto sobre la recta AC, tal que BC = BD = DA.Determine la medida del angulo ∠ABD, si:

a) D esta entre A y C.

b) A esta entre D y C.

12. En un 4ABC, D es un punto sobre el lado AC tal que AB = AD. Si ∠ABC−∠ACB =90°, hallar el ∠CBD.

13. En la figura 17, el ∠ABC = ∠ACE, DC = EC, ¿Que lınea notable es AD del 4BCA?

Figura 17

14. Se tiene un triangulo isosceles ABC, AB = BC en el cual se traza al altura AF tal queBF = 6 y FC = 2. Hallar AC.

15. ¿Cual es el valor de b− a en la figura 18?

Figura 18

13

Page 14: Geometria Plana

16. La hipotenusa BC de un triangulo rectangulo ABC se divide en 4 segmentos congruentespor los puntos G, E y H. Si BC = 20, encuentra la suma de los cuadrados de las longitudesde los segmentos AG, AE y AH. Figura 19.

Figura 19

17. (*) Dado un cuadrado ABCD, se construyen los triangulos equilateros ABP (exterior-mente) y ADQ (interiormente). Probar que C, P y Q estan alineados.

18. (*) Sea ABC un triangulo rectangulo con ∠CAB = 90°. D es un punto sobre la prolon-gacion de BC tal que BD = BA. E es un punto en el mismo semiplano que A respectode BC, tal que CE ⊥ BC y ademas CE = CA. Mostrar que A,D y E estan alineados.

19. El cuadrilatero ABCD mostrado en la figura 20 cumple que AB ‖ CD y BC ‖ DA.6

Sobre las prolongaciones de AB y AD se construyen puntos E y F tales que BC = BEy DC = DF . Demuestre que C, E y F estan alinedos.

Figura 20

20. (*) En la figura adjunta, AB = BC = CD =DE = EF = FG = GA. Calcule la medida del∠DAE.

6El cuadrilatero ABCD es un paralelogramo.

14

Page 15: Geometria Plana

21. (*) (XXVIII Olimpiada Brasilena de Matematica) En la figura 21, AB = AC, AM = ANy ∠CAM = 30°, encuentre el valor del ∠BMN .

Figura 21

22. Los lados de un triangulo isosceles son 12 y 5 metros, ¿cual es su perımetro?

23. Muestre que los lados de un triangulo cumplen que |a− b| < c y que c < a+b+c2

.

24. Muestre que es posible construir un triangulo con segmentos de longitudes a, b, c si y soloexisten numeros positivos x, y, z tales que: a = x+ y, b = y + z, c = z + x.

Problemas de Refuerzo.

25. (*) (Etapa semifinal Estatal de XXII Olimpiada Mexicana de Matematicas) En la figura22 se muestra un hexagono regular ABCDEF de lado 1. Los arcos del cırculo que estandibujados tienen centro en cada vertice del hexagono y radio igual a la distancia al verticeopuesto. P , Q, R, S, T y U son los puntos de corte de estos arcos. ¿Cuanto mide cadalado del hexagono PQRSTU?

Figura 22

15

Page 16: Geometria Plana

3. Congruencia de Triangulos.

CRITERIOS DE CONGRUENCIA.

Definicion de Congruencia de triangulos. El 4ABC es congruente al 4A′B′C ′ si: AB =A′B′, AC = A′C ′, BC = B′C ′, ∠ABC = ∠A′B′C ′, ∠ACB = ∠A′C ′B′ y ∠BAC = ∠B′A′C ′.Simbolicamente: 4ABC = 4A′B′C ′. Vease figura 23.

Figura 23: Definicion de Igualdad de Triangulos.

La definicion anterior establece que dos triangulos son congruentes si tanto los lados comolos angulos se presentan en pares respectivos congruentes. Esto, segun la vision de Euclides,significa que un triangulo es posible superponerlo sobre el otro (se puede desplazar, girar oreflejar) y coincidira de manera perfecta. Sin embargo, es importante mencionar que en muyraras ocasiones se tendra a disposicion tanta informacion, de allı la importancia de los criteriosde congruencia, que establecen los requisitos mınimos para garantizar que dos triangulos soncongruentes.

El siguiente es el primero de los tres criterios de congruencia de triangulos, y se denominacriterio de LADO-ANGULO-LADO, en sımbolos: L-A-L.

Criterio L-A-L. Si los triangulos ABC y A′B′C ′ presentan las congruencias: AB = A′B′,AC = A′C ′ y ∠BAC = ∠B′A′C ′, entonces 4ABC = 4A′B′C ′.

Figura 24: Criterio LAL

Segun el criterio L-A-L, dos triangulos son congruentes si en uno de ellos existen dos lados y elangulo (comprendido entre dichos lados), respectivamente congruentes a dos lados y el angulo(comprendido entre dichos lados), en el otro triangulo.

Criterio A-L-A. Sean ABC y A′B′C ′ dos triangulos tales que: AC = A′C ′, ∠BCA = ∠B′C ′A′

y ∠BAC = ∠B′A′C ′, entonces 4ABC = 4A′B′C ′.

16

Page 17: Geometria Plana

Figura 25: Criterio ALA.

Criterio L-L-L. Si un triangulo tiene sus tres lados respectivamente congruentes a los treslados de otro triangulo, entonces estos dos triangulos son congruentes.

Figura 26: Criterio LLL.

Ahora demostraremos el Recıproco del Teorema de Pitagoras.Demostracion: Sea ABC un triangulo talque BC2 = AB2 + AC2, por construccion sea el4A′B′C ′ rectangulo en A′ tal que A′B′ = AB y A′C ′ = AC, entonces por el teorema dePitagoras B′C ′2 = A′B′2 + A′C ′2, ası que B′C ′2 = BC2, de donde B′C ′ = BC y por el criterioLLL, se deduce que el 4A′B′C ′ = 4ABC, por lo tanto el ∠BAC = ∠B′A′C ′ = 90°.

TEOREMA DE LA BASE MEDIA

En todo triangulo, el segmento que une los puntos medios de dos lados es paralelo al tercerlado e igual a su mitad.

En la figura 27, MN es el segmento que une los puntos medios de los lados AB y BC del4ABC,

a este segmento se le llama BASE MEDIA DEL TRIANGULO. Se verifica que MN =AC

2y

que MN ‖ AC.Demostracion:

1. Prolongar el segmento MN hasta el punto P tal que MN = NP .

2. Los triangulos MNB y PNC son congruentes, ya que BN = NC, MN = NP y el∠PNC = ∠BNM , por consiguiente, el ∠NCP = ∠MBN , por lo tanto, CP ‖MB (Porangulos alternos internos iguales). Ademas, PC = MB = MA; con lo cual se tiene que:MA = PC.

17

Page 18: Geometria Plana

Figura 27: Teorema de La Base Media.

Figura 28: Menor Media en un Triangulo Rectangulo.

3. Uniendo el punto A con el punto P se forman los triangulos congruentes AMP y ACP(por L A L) ya que MA = PC, AP = AP , ∠MAP = ∠APC (por angulos alternosinternos entre las paralelas MA y PC). Luego, MP = AC, entonces NP = 1

2MP = 1

2AC.

Ademas, ∠PAC = ∠MPA, de donde MP ‖ AC o que MN ‖ AC.

Corolario: Menor mediana de un triangulo rectangulo. En todo triangulo rectangulo,la mediana relativa a la hipotenusa es la mitad de la longitud de la hipotenusa y es la menorde las tres medianas del triangulo.

Demostracion: En la figura 28, BM es la mediana relativa a la hipotenusa AC del 4ABC,probaremos que BM = AC

2; (con lo cual se tendra que BM = AM = MC). Si por M se traza

una paralela al lado AB, que corte al lado BC en N , entonces N es el punto medio de BC yel ∠MNC = 90°, los triangulos BNM y CNM son congruentes por el criterio L-A-L, luegoMB = MC = AM .Probar que BM es la menor mediana (Ejercicio).

18

Page 19: Geometria Plana

Ejercicios.

1. (*) En la figura adjunta, ABC es un triangulo equilateroy CDEF es un cuadrado. Se construye un punto G talque CF = CG y ademas ∠CFG = 15°. Probar que∠AGC = ∠BDC.

2. Dado un triangulo equilatero ABC, se construye un triangulo equilatero DEF cuyosvertices estan sobre los lados del 4ABC, tal como muestra la figura 29. Demuestre quelos triangulos ADF , BED, CFE son todos congruentes entre si.

Figura 29

3. ABCD es un cuadrado, E, F , G y H son puntos sobre los lados AB, BC, CD, DA, res-pectivamente, tal que EFGH tambien es cuadrado. Demuestre que los triangulos AEH,BFE, CGF , DHG son todos congruentes entre si. Figura 30.

Figura 30

19

Page 20: Geometria Plana

4. ABCDE y FGHIJ son pentagonos regulares (Vease figura 31). Demuestre que lostriangulos AFJ , BGF , CHG, DIH, EJI son todos congruentes entre si.

Figura 31

5. Si AB ‖ CD y AB = CD entonces, AD = BC y AD ‖ BC7.

6. Demuestre que dos triangulos desplazadosson congruentes. Sugencia: Utilice el pro-blema anterior.

7. Demuestre que dos triangulos rotados son congruen-tes.

7El cuadrilatero ABCD se denomina paralelogramo.

20

Page 21: Geometria Plana

8. Demuestre que dos triangulos reflejados con respecto a un punto 8 son congruentes.

9. Demuestre que dos triangulos reflejados con respecto auna recta son congruentes.

Importante: Las traslaciones, rotaciones y reflexiones no cambian el tamano ni la formade un triangulo.

10. (*) En la figura adjunta, ABCD un cuadrado y EF ⊥ GH.Demuestre que que EF = GH.

11. Dos cuadrados ABCD y EHGF , ambos de lado l,estan colocados en manera tal que un vertice de unoesta en el centro del otro (como en la figura anexa).

Demuestre que el area del cuadrilatero EJBK esl2

4y por ende no depende de la posicion de J (o K).

8La reflexion con respecto a un punto es equivalente a una rotacion de 180°

21

Page 22: Geometria Plana

12. En un 4ABC el ∠B = 2∠C, la mediatriz del lado AC corta en F al lado BC. HallarAB, si BF = 3 y FC = 9.

13. (*) (Examen final de XVI Olimpiada mexicana de Matematica) Los angulos de un triangu-lo ABC estan en progresion aritmetica (∠B − ∠A = ∠C − ∠B = θ), D, E, y F son lospuntos medios de los lados BC, CA y AB, respectivamente. Llamamos H al pie de laaltura trazada desde C (que cae entre B y F ) y G a la interseccion entre DH y EF .¿Cuanto vale ∠FGH?

14. En la figura 32, AC = 12 AF = 4 y ∠BAF = 30. Hallar BF si AG = GC.

Figura 32

15. En la figura 33, AG = GC, el ∠AFG = 20°. Hallar el ∠FAC, si AC = 2BF .

Figura 33

16. (*) Sea ABCD un cuadrado. Se construyen triangulos equilateros ADP y ABQ como semuestra en la figura 34. Sea M la interseccion de CQ con AD y N la interseccion de CPcon AB. Demuestre que CMN es un triangulo equilatero.

22

Page 23: Geometria Plana

Figura 34

Problemas de Refuerzo.

17. En la figura 35, ABC, CDE y EFA son triangulos isosceles, con el ∠ABC = ∠CDE =∠EFA = 120°. Probar que el 4BDF es equilatero.

Figura 35

18. (*) 4ABC es un triangulo isosceles con ∠ABC = ∠ACB = 80°. D es un punto en ACtal que ∠ABD = 10°. Demuestre que AD = BC.

23

Page 24: Geometria Plana

4. Cuadrilateros: Clasificacion y Propiedades.

CLASIFICACION.

Los cuadrilateros pueden clasificarse de acuerdo a sus diagonales de la siguiente forma:

Cuadrilatero Convexo: Es un cuadrilatero con las dos diagonales en su interior.

Cuadrilatero Entrante: Es un cuadrilatero con una diagonal en el interior y otra en el exte-rior.

Cuadrilatero Cruzado Es un cuadrilatero con las diagonales en su exterior.9

Es muy frecuente que se considere que un cuadrilatero es convexo, a menos que se especifique locontrario. Esto es ası porque muchos resultados son mas claros en un cuadrilatero convexo, sinembargo, es importante darse cuenta que existen teoremas que no se cumplen para cualquiertipo de cuadrilateros, por ejemplo:

Teorema: La suma de los angulos internos de un cuadrilatero no cruzado es 360.

La demostracion de este resultado se basa en la diseccion del cuadrilatero en dos triangu-los cuyos angulos internos conforman los angulos internos del cuadrilatero, sin embargo, estascondiciones no pueden lograrse en un cuadrilatero cruzado; de hecho, la suma de los angulosinternos puede hacerse arbitrariamente pequena cuando el cuadrilatero es cruzado.

Tambien hay otras clasificaciones de cuadrilateros de acuerdo a sus lados y angulos.

Cuadrilatero Equiangulo: un cuadrilatero (convexo) es equiangulo si todos sus angulos inter-nos son iguales; dado el teorema anterior, los angulos son iguales a 90, por ello este cuadrilateroes llamado rectangulo.

Cuadrilatero Equilatero: un cuadrilatero (convexo) es equilatero si todos sus lados son igua-les. A este cuadiratero tambien se le conoce como rombo.

Cuadrado: es un cuadrilatero que es equiangulo y equilatero.

Paralelogramo: es un cuadrilatero con los lados opuestos paralelos.

Trapecio: es un cuadrilatero con un par de lados opuestos paralelos.10

9Tanto los cuadrilateros convexos como los entrantes son cuadrilateros simples, que son los cuadrilateroscuyos lados no se cortan salvo en los extrenos; en contraposicion, los cuadrilateros cruzados no son simples.

10Note que un paralelogramo es tambien un trapecio.

24

Page 25: Geometria Plana

PARALELOGRAMOS

Dado el paralogramo ABCD, por propiedades de angulos entre paralelas es posible probar elsiguiente resultado:

Teorema: Los angulos opuestos son iguales y los angulos consecutivos son suplementarios:∠ABC = ∠CDA = θ y ∠BCD = ∠DAB = 180− θ.

Por otra parte, por criterio ALA, 4ABC ≡ 4CDA; esto implica que AB = CD y BC = DA,i.e.

Teorema: Los lados opuestos de un paralogramos son iguales.

A partir de esto, si M es la interseccion de AC con BD, por criterio ALA, 4ABM ≡ 4CDM ,por lo que AM = CM y BM = DM , i.e.

Teorema: Las diagonales de un paralelogramo se bisecan.

Ademas, se cumple un resultado sofisticado y muy importante:

Teorema: Ley del Paralelogramo. Si ABCD es un paralelogramo entonces el doble de lasuma de los cuadrados de los lados es igual a la suma de los cuadrados de las diagonales, esdecir

2(AB2 +BC2

)= AC2 +BD2

Demostracion: Aplicando la Ley del Coseno a 4ABC y 4ABD se tiene

AC2 = AB2 +BC2 − AB ·BC cos θ

DB2 = AB2 + AD2 − AB · AD cos(180− θ)⇒ AC2 +DB2 = 2

(AB2 +BC2

)− AB ·BC (cos θ + cos(180− θ))

y dado que cos θ = − cos(180− θ) el resultado se sigue inmediatamente.

RECTANGULOS

En primer lugar, es importante notar que todo rectangulo es paralelogramo (por angulos en-tre paralelas), por lo que todos los resultados probados anteriormente son heredados a todorectangulo; pero los rectangulos tienen propiedades adicionales:

Observe que por criterio LAL, 4ABC ≡ 4ABD, por lo que AC = BD y entoncesTeorema: Las diagonales de un paralelogramo son iguales; ademas, el punto de interseccionde estas equidista de los cuatro vertices y por tanto es el centro de una circunferencia que pasapor todos los vertices.

Por otra parte, observe que si se aplica la ley del paralelogramo a un rectangulo se obtiene elTeorema de Pitagoras.

25

Page 26: Geometria Plana

ROMBOS

Dado un rombo ABCD, por criterio LLL, 4ABC ≡ 4CDA, y por lo tanto ∠BAC = ∠DACy ∠BCA = ∠DAC, lo cual implica BC ‖ AD y AB ‖ CD, i.e., todo rombo ABCD es unparalelogramo. Ademas, por las mismas congruencias se tiene

Teorema: Las diagonales de un rombo cumplen ser una mediatriz de la otra.

Teorema: Las diagonales de un rombo bisecan a los angulos interiores del rombo; esto implicaque el punto de corte de las diagonales equidista de los cuatro lados del rombo y es el centrode una circunferencia tangente a estos.

TRAPECIOS

Dado el trapecio ABCD (con AB ‖ CD), se construyen los puntos medios de BC y DA, M yN , respectivamente. Si el cuadrilatero MNAB se rota con centro en M y angulo 180 se generaun cuadrilatero MN ′A′C; observe que ND = N ′A′ y ND ‖ N ′A′, por lo que DNN ′A′ es unparalelogramo y

NN ′ = DA′

2MN = DC + CA′

2MN = DC + AB

⇒MN =AB + CD

2

El segmento MN es llamado base media del trapecio, y por lo recien demostrado se tiene

Teorema: La base media de un trapecio es igual a la semisuma de las bases.

Por otra parte, hay ciertos trapecios que reciben nombres particulares; el trapecio rectangulo esaquel que las bases son perpendiculares a alguno de los otros lados; y por otra parte, el trapecioisosceles es aquel que los lados (distintos de las bases) tienen igual longitud. 11

Ejercicios

1. Dado el trapecio ABCD con AB ‖ CD, demuestre que la bisectriz interior del ∠A esparalela a la bisectriz exterior del ∠D.

2. A un rombo ABCD se le construyen exteriormente los cuadrados ABEF y BCGH.Demuestre que 4ABD = 4EBH.

3. (*) Sea ABCD un paralelogramo. Se construyen triangulos equilateros exteriores 4CDPy 4ADQ. Demuestre que el 4BPQ es equilatero.

4. Demuestre que las bisectrices interiores de un paralelogramo forman un rectangulo (¿que su-cede si el paralelogramo es ademas rombo?).

11Los trapecios isosceles son muy importantes cuando se estudian los angulos en la circunferencia; resulta queun trapecio es isosceles si y solo si los cuatro vertices se ubican sobre una misma circunferencia.

26

Page 27: Geometria Plana

5. Demuestre que las bisectrices exteriores de un paralelogramo forman un rectangulo.

6. Sea ABCD un paralelogramo. La bisectriz interna del ∠CDA corta a BA en M , y labisectriz interna del ∠BAD corta a CD en N . Demuestre que ADNM es un rombo.

7. Demuestre que si por el punto de interseccion de las diagonales de un rombo se tra-zan perpendiculares a los lados del rombo, entonces los puntos de interseccion de dichasperpendiculares con los lados del rombo forman un rectangulo.

8. Demuestre que las bisectrices de los angulos definidos por las diagonales de un rombo,cortan a los lados del rombo en cuatro puntos que forman un cuadrado.

9. En un4ABC seaG la interseccion de las medianasBB′ y CC ′. SeanB′′, C ′′ las reflexionesde G respectivas a los puntos B′ y C ′.

a) Demuestre que AGCB′′ y AGBC ′′ son paralelogramos.

b) A partir de lo anterior, demuestre que BCB′′C ′′ tambien es paralelogramo.

c) Demuestre que A′ pertenece a la recta AG, y concluya que las tres medianas de untriangulo concurren en el punto G, llamado el centroide del 4ABC.

d) Demuestre que CG = 2GC ′; relaciones similares se cumplen para las otras dos media-nas.

10. Teorema de Varignon: Dado un cuadrilatero ABCD (no necesariamente convexo), seconstruyen los puntos medios L, M , N , O, P , Q, de los segmentos de recta AB, BC, CD,DA, BD, AC, respectivamente. Figura 36.

a) Demuestre que LMNO, LPNQ, OPMQ, son paralelogramos.

b) Demuestre que LN , OM , PQ concurren en un punto, llamado el centroide del cua-drilatero ABCD.

c) Demuestre que el perımetro de LMNO es igual a AC + BD; resultados similares secumplen para los otros paralelogramos.

Figura 36: Teorema de Varignon

27

Page 28: Geometria Plana

11. Sea ABCD un paralelogramo tal que existe un punto E sobre el lado AB que cumple∠CED = 90. Sean M y N los pies de las perpendiculares trazadas desde A y B haciaDE y CE, respectivamente. Demuestre que AC, BD y MN concurren.

12. (*) (Hector Alberti) Sea ABCD un cuadrado. Se construyen los triangulos equilaterosBDA′, ACB′, BDC ′ y ACD′. Demuestre que el A′B′C ′D′ es tambien un cuadrado.

13. (*) (II Olimpiada Matematica del Cono Sur) En la figura 37 ABCD y AECF son para-lelogramos. Demuestre que BEDF es paralelogramo.

Figura 37

Problemas de Refuerzo.

14. (*) ABCD es un cuadrilatero convexo y O es un punto en su interior. Sean P , Q, R, S,los puntos medios de los lados AB, BC, CD, DA, respectivamente. Por P se traza unaparalela a OR, por Q se traza una paralela a OS, por R se traza una paralela a OP , ypor S se traza una paralela a OQ. Demuestre que estas cuatro rectas concurren.

15. (*) Un trapecio isosceles tiene diagonales perpendiculares y su area es 2010, determine sualtura.

16. (*) (IX Competencia de Clubes Cabri, Segunda Ronda) Sea ABCDEF un hexagonoregular cuyo centro es O. Se construyen los cuadrados FSOP y ORCQ. Demuestre queAPQB y SEDR son rectangulos. Figura 38.

17. (*) Sobre los lados del 4ABC se trazan exteriormente los cuadrados ABPQ, CARS yBCTU . Luego se trazan los paralelogramos AQA′R, CSC ′T y BUB′P .

a) Sean A′′, B′′, C ′′ los centros de los cuadrados BCTU , CARS, ABPQ, respectiva-mente. Demuestre que estos centros estan sobre los lados del 4A′B′C ′.

b) Demuestre que AA′′, BB′′, CC ′′ concurren.

28

Page 29: Geometria Plana

Figura 38

18. (*) Se dibujan cuadrados exteriores a los lados de un paralelogramo, demuestre que:

a) El cuadrilatero determinado por los centros de esos cuadrados es un cuadrado.

b) Las diagonales de ese cuadrado son concurrentes con las del paralelogramo.

19. (*) Dado un 4ABC, se construyen exteriormente los triangulos rectangulo isosceles4ACP y 4BCQ, con AC y BC como hipotenusas. Si M es el punto medio de AB,demuestre que el 4MPQ tambien es un triangulo rectangulo isosceles.

29

Page 30: Geometria Plana

5. Angulos en la Circunferencia.

LA CIRCUNFERENCIA Y SUS ELEMENTOS

Una circunferencia es el lugar geometrico de puntos que equidistan de un punto dado, llamadoel centro de la circunferencia; la distancia de cada punto de la circunferencia al centro es el radio.

Por otra parte, todos los puntos que estan a una distancia del centro menor o igual al radioforman el cırculo; estos puntos quedan “al interior” o sobre la circunferencia.

Si A y B son dos puntos de una circunferencia, el segmento de recta AB define una cuerda; enparticular, si el centro de la circunferencia pertenece a la cuerda, esta es llamada diametro. Esimportante mencionar que para cada punto de la circunferencia existe exactamente un puntodiametralmente opuesto.

En la figura 39, se tiene una circunferencia de centro O y radio r = OA = OB = OA′; ABy AA′ son cuerdas, pero AA′ es tambien diametro, i.e, A′ es diametralmente opuesto a A yviceversa. Observe que por la desigualdad triangular aplicada al triangulo isosceles 4AOB

Figura 39

AB < AO +BO

= r + r

= AA′

Si A es un punto fijo, esta desigualdad es valida para cualquier punto B sobre la circunferencia(excepto cuando B = A′ lo cual implica AB = AA′). Esto quiere decir que el diametro es lamayor de todas las cuerdas.A las porciones de circunferencia que quedan entre dos puntos ubicados en la circunferencia,se les llama arcos de circunferencia; note que dos puntos sobre una circunferencia definen dosarcos de circunferencia. Tambien, si un angulo tiene vertice sobre el centro de la circunferencia yesta formado por dos radios, sera llamado angulo central ; de nuevo, ∠AOB hace referencia a dosangulos, cuya suma es 360, y subtienden respectivamente a uno de los arcos AB. Finalmente,si un angulo tiene el vertice sobre la circunferencia y esta formado por dos cuerdas, sera llama-do angulo inscrito; en la figura anterior, ∠AA′B es un angulo inscrito que subtiende al arco AB.

30

Page 31: Geometria Plana

Teorema: El angulo central es el doble del angulo inscrito que subtiende el mismo arco.

Demostracion: Considere la figura 40, se demostrara que ∠AOB = 2∠APB en los tres casosmostrados. En la circunferencia de la izquierda, sea P ′ el punto diametralmente opuesto a P ;observe que 4APO y 4BPO son triangulos isosceles, y por el teorema del angulo externo setiene

∠AOB = ∠AOP ′ + ∠BOP ′

= (∠APO + ∠OAP ) + (∠BPO + ∠OBP )

= 2∠APO + 2∠BPO

= 2 (∠APO + ∠BPO)

= 2∠APB

Figura 40

El caso de la circunferencia del medio es mas sencillo y se deja como ejercicio para el lector.Para la circunferencia de la derecha, el trabajo es analogo y solo cambia en un pequeno arregloalgebraico

∠AOB = ∠BOP ′ − ∠AOP ′

= (∠BPO + ∠OBP )− (∠APO + ∠OAP )

= 2∠BPO − 2∠APO

= 2 (∠BPO − ∠APO)

= 2∠APB

Corolario: Todos los angulos inscritos que subtienden el mismo arco son iguales (Ver figura41). En particular, los angulos internos son iguales a 90° si subtienden a una semicircunferencia.

Demostracion: Todos los angulos mostrados en la figura 41 son iguales a la mitad del ∠AOB,y por tanto, son iguales entre sı. En particular, si AB fuera un diametro, ∠AOB = 180° y portanto ∠APB = 90°. 12

Hay un par de angulos mas que son importantes: Si un punto P es interno a la circunferencia,el angulo de vertice P formado por dos cuerdas que pasan por P se llama angulo interior. De

12Observe que en cualquier triangulo rectangulo, el punto medio de la hipotenusa equidista de los tres vertices.

31

Page 32: Geometria Plana

Figura 41

forma similar, si P es exterior y dos cuerdas de la circunferencia (al prolongarse) pasan por P ,el angulo con vertice P es llamado angulo exterior.

Dejamos como ejercicio demostrar el siguiente teorema:

Teorema: Los angulos interior y exterior mostrados en la figura 42 cumplen las formulassiguientes:

∠AQC =∠BOD + ∠AOC

2

∠APC =∠BOD − ∠AOC

2

Figura 42

CUADRILATEROS CICLICOS

Ahora suponga que sobre una circunferencia se ubican cuatro puntos A, B, C, D, como semuestra en la figura 43. Al cuadrilatero ABCD se le llama cuadrilatero cıclico o concıclico.Observe que

∠ABC + ∠CDA =α

2+β

2= 180◦.

32

Page 33: Geometria Plana

Figura 43

Y analogamente ∠DAB+∠BCD = 180°. Esto significa que si ABCD es un cuadrilatero cıclicoy convexo, entonces los angulos opuestos son suplementarios. Tambien, es posible demostrar porcontradiccion el recıproco de este resultado: si suponemos que ABCD es tal que ∠B+∠D = 180pero no es cıclico, se define el punto D′ como la otra interseccion de AD con el circuncırculodel 4ABC, y como ABCD′ es cıclico (por construccion) entonces ∠B + ∠D′ = 180, luego,∠D = ∠D′, lo cual implica la contradiccion CD ‖ CD′ (rectas paralelas que se cortan en C).Ası, se ha demostrado el siguiente teorema:

Teorema: El cuadrilatero convexo ABCD es un cuadrilatero cıclico si y solo si

∠A+ ∠C = 180◦ = ∠B + ∠D

Tambien, otro criterio muy util y cuya demostracion tambien se basa en el corolario anterior es

Teorema: El cuadrilatero convexo ABCD es un cuadrilatero cıclico si y solo si se cumplealguna de las siguientes igualdades

∠ABD = ∠ACD

∠BCA = ∠BDA

∠BAC = ∠BDC

∠CAD = ∠CBD

Es importante recalcar que NO todo cuadrilatero puede ser inscrito en una circunferencia; porejemplo, un paralelogramo no sera cıclico a menos que sea rectangulo.

RECTAS Y CIRCUNFERENCIAS TANGENTES A UNA CIRCUN-FERENCIA

Dada una circunferencia, una recta puede ser tangente o secante a la circunferencia, depen-diendo si la corta en uno o dos puntos, respectivamente; en cualquier otro caso, se dice que la

33

Page 34: Geometria Plana

recta no corta a la circunferencia.13

Sea l una recta secante a la circunferencia que corta a la circunferencia en A y B (A 6= B);como el 4AOB es isosceles, ∠OAB < 90. Recıprocamente, si por A se traza una recta l talque uno de los angulos que forma con OA es menor que 90, se puede construir un punto Bsobre l tal que ∠OAB = ∠ABO < 90 y A 6= B (basta proyectar O sobre l y luego reflejar Acon respecto a este punto, el resultante es el punto B); entonces el 4AOB es isosceles, por loque OA = r = OB, i.e. B pertenece a la circunferencia y por tanto l corta a la circunferenciaen dos puntos distintos. Ası

Teorema: Una recta l corta a una circunferencia de centro O en dos puntos distintos A y B siy solo si un angulo entre l y OA es agudo.

Corolario: Si l es una recta tangente en A a una circunferencia de centro O, ninguno de losangulos entre l y OA puede ser agudo, y por tanto l ⊥ OA.

A partir de este resultado se prueban otros resultados muy conocidos y utiles, que dejamos deejercicios para el lector.

Teorema: Dado un punto P externo a una circunferencia de centro O, si PA y PB son segmen-tos tangentes a la circunferencia en A y B, respectivamente, entonces el cuadrilatero PAOB escıclico y bisosceles.

Corolario: Dado un punto P externo a una circunferencia de centro O, la circunferencia dediametro PO corta a la circunferencia dada en dos puntos A y B tales que PA y PB son rectastangentes.

Definicion: El angulo semi-inscrito en una circunferencia es aquel que se forma con una cuerday la recta tangente en alguno de los extremos de la cuerda.

Teorema: La media del angulo semi-inscrito definido por la cuerda AB es igual a la medidade un angulo inscrito que subtiende al arco AB.

Demostracion: Considere la figura 44. Como APBO es cıclico, entonces ∠PAB = ∠POB;ademas, como PO es la mediatiz de AB, ∠POB = ∠POA, por lo que

∠PAB =∠AOB

2= ∠AQB

Por otra parte, dada una circunferencia, otra circunferencia puede ser secante o tangente a laprimera, dependiendo si la corta en uno o dos puntos, respectivamente; en cualquier otro casose dice que las circunferencias no se cortan.14

13Cuando la recta es tangente a la circunferencia puede considerarse como un caso muy peculiar en el cuallos “dos” puntos de corte coinciden.

14Tambien aca puede considerarse a las circunferencias tangentes como un caso especial de circunferenciassecantes en el cual los puntos de corte coinciden.

34

Page 35: Geometria Plana

Figura 44

Ademas, dos circunferencias pueden posicionarse una dentro de la otra, y claramente, la cir-cunferencia de radio mayor es la externa mientras que otra es la interna; particularmente, si lascircunferencias tienen el mismo centro se llaman concentricas. Finalmente, combinando estasdefinciones se tienen las circunferencias tangentes exteriormente y las tangentes interiormente.

Teorema: Dadas dos circunferencias de centros O1 y O2 que se cortan en dos puntos distintosA y B, se cumple que O1O2 ⊥ AB.

Teorema: Si dos circunferencias de centros O1 y O2 son tangentes en A, se cumple que O1, Ay O2 estan alineados.

Teorema:

a) Dos circunferencias, una dentro de la otra, no tienen rectas tangentes en comun.

b) Dos circunferencias tangentes interiormente tienen una recta tangente comun.

c) Dos circunferencias secantes (en dos puntos distintos) tienen dos rectas tangentes en comun.

d) Dos circunferencias tangentes exteriormente tienen tres rectas tangentes en comun.

e) Dos circunferencias no secantes y tal que ninguna contiene a la otra, tienen cuatro rectastangentes en comun.

Ejercicios

1. Si el ∠MPQ = 20, determine el valor del ∠QON en la figuraadjunta.

35

Page 36: Geometria Plana

2. Dado un angulo inscrito BAC, y su angulo central BOC, se sabe que ∠BAC+∠BOC =180°. Calcular el ∠OBC.

3. En la figura 45, BCDO es un rombo. Determine el valor del angulo θ y la medida de lasdiagonales de BCDO si el radio de la circunferencia mide 6.

Figura 45

4. Un cuadrilatero cıclico ABCD satisface ∠ABC = 2∠CDA = θ. Calcule θ.

5. En la figura 46, PR es una tangente comun. Calcule el valor del ∠PQR.

Figura 46

6. En la figura adjunta, el ∠AFE = 100° y el ∠BCD =150°. Calcule el ∠AGB.

7. Dado un angulo ∠AOB, se trazan dos rectas l ⊥ OA y m ⊥ OB. Si P es el punto decorte de l y m, demuestre que A, B, O, P se ubican sobre una misma circunferencia.

36

Page 37: Geometria Plana

8. Las bisectrices BP y CQ del 4ABC se cortan en I. Demuestre que si ∠BAC = 60entonces 4PQI es isosceles.

9. En la figura 47 se ha tomado un punto C sobre la circunferencia; AC y BC cortan a lasegunda circunferencia en D y E respectivamente. Probar que OC ⊥ DE.

Figura 47

10. (*) Dada la figura 48, demuestre que AB ‖ A′B′.

Figura 48

11. En la figura 49 CR es una recta tangente en C, demuestre que AB ‖ CR.

Figura 49

37

Page 38: Geometria Plana

12. Dos circunferencias Γ1 y Γ2 son tangentes (interior o exteriormente) en P (Ver figura 50).Dos rectas que pasan por P cortan a Γ1 y Γ2 en A y C, y en B y D, respectivamente.Demuestre que AB ‖ CD.

Figura 50

13. (*) Dos circunferencias de centros O1 y O2 son tangentes (interna o externamente) en unpunto P ; por este punto se traza una recta que corta nuevamente a la circunferencias enA y B, respectivamente. Demuestre que AO1 ‖ BO2.

14. Dos circunferencias son tangentes externamente en el punto A. Una tangente exteriorcomun toca a una circunferencia en B y a la otra en C. Demostrar que ∠BAC = 90°.

15. En la figura 51, DE es tangente en D, y C es el punto medio del arco AD. Encuentre elvalor del angulo seminscrito ADE.

Figura 51

38

Page 39: Geometria Plana

16. Determine el valor del ∠DCF , sabiendo BE es tangente en el punto D a la circunferenciade centro O. Ver Figura 52.

Figura 52

17. Si el ∠AEB = 30, ∠ADE = 20 y ∠ACE = 35, calcule el ∠AFB. Vease figura 53.

Figura 53

18. Dada una circunferencia de diametro BC, se toma un punto P en la prolongacion de BC,y se traza la tangente AP . Si AP = AB y O es el centro de la circunferencia, demuestreque el 4AOC es equilatero.

19. (*) Dadas dos circunferencias una fuera de la otra, demuestre que las tangentes comunesexternas forman segmentos iguales; analogamente, las tangentes comunes internas formansegmentos iguales.

20. (*) Teorema de Pithot. Demuestre que en todo cuadrilatero inscribible, la suma delados opuestos es igual.

21. (*) Teorema de Steiner. En todo cuadrilatero exinscrito a una circunferencia, la dife-rencia de las longitudes de lados opuestos es igual.

39

Page 40: Geometria Plana

22. Demuestre que las mediatrices de un cuadrilatero son concurrentes si y solo si es cıclico.

23. Demuestre que el cuadrilatero convexo ABCD es inscribible si y solo si los incırculosrespectivos del 4ABC y 4CDA son tangentes.

24. Demuestre que las bisectrices internas de un cuadrilatero son concurrentes si y solo si esinscribible.

25. Demuestre que todo rombo es inscribible.

26. En la figura 54, AB es una cuerda y por D se traza una recta tangente a la circunferenciaparalela a AB. Demuestre que CD es bisectriz del ∠ACB.

Figura 54

27. Determine las medidas de ∠ACB y ∠ACO de la figura 55.

Figura 55

28. Cuatro cilindros de diametro 1 estan pegados apretadamente poruna cuerda muy fina, como en la figura adjunta. Demostrar que lacuerda tine longitud 4 + π. Demostrar tambien que el area som-breada entre los cilindros es 1− π

4.

29. En la figura 56, ABCD es un trapecio isosceles con AB ‖ CD y DA = BC = 2; tomandoDA y BC como diametros, se construyen dos circunferencias tangentes. Si DC = 3AB,calcule el area del trapecio.

40

Page 41: Geometria Plana

Figura 56

30. La figura 57 esta formada por un paralelogramo y dos circunferencia tangentes entre sı ytangentes a tres lados del paralelogramo. Sabiendo que el radio de las mismas mide lacuarta parte del lado menor del paralelogramo, calcule la razon entre el lado mayor delparalelogramo y el radio de las circunferencias.

Figura 57

31. En la figura 58, ABCDEF es un hexagono regular y las circunferencias de centro en losvertices son tangentes dos a dos. Si las circunferencias sobre los vertices B, D, F soniguales, demuestre que las circunferencias restantes son iguales.

Figura 58

32. Alrededor de una circunferencia se construyen diez circunferencias tangentes a la originaly tangentes entre sı (Vease figura 59). Demuestre que la suma de las areas de las diezcircunferencias es el doble del area de la circunferencia mayor.

41

Page 42: Geometria Plana

Figura 59

33. (*) Teorema de Miquel: Dado un 4ABC, sean X, Y , Z puntos sobre AB, BC, CA,respectivamente. Demuestre que los circuncırculos de 4AXZ, 4BYX, 4CZY tienen unpunto en comun M .

34. (X OMCC - P2, Aaron) Sea ABCD un cuadrilatero concıclico con diametro AC, y seaO el centro de su circunferencia. Se construyen los paralelogramos DAOE y BCOF .Demuestre que si E y F estan sobre la circunferencia entonces ABCD es rectangulo.

35. (*) Sea ABC un triangulo, y sean L y N las intersecciones de la bisectriz del angulo Acon el lado BC y el circuncırculo de ABC respectivamente. Construimos la interseccionM del circuncırculo de ABL con el segmento AC. Prueba que los triangulos BMN yBMC tienen la misma area.

36. (*) Sea AB el diametro de una semicircunferencia. Se colocan los puntos M y K sobrela semicircunferencia y sobre AB, respectivamente.15 Sea P el centro de la circunferenciaque pasa por A, K y M ; sea Q el centro de la circunferencia que pasa por B, K y M .Demuestre que MPKQ es concıclico.

37. (*) Las circunferencias Γ1 y Γ2 se cortan en los puntos A y B. Por el punto A se trazauna recta que corta nuevamente a las circunferencias Γ1 y Γ2 en los puntos C y D,respectivamente. Por los puntos C y D se trazan tangentes a las circunferencias, lascuales se cortan en el punto M . Demuestra que MCBD es cıclico.

38. (*) El 4ABC cumple que ∠A = 90° y AB = AC. Se toma un punto E del segmentoAB, se construye interiormente un triangulo equilatero AEF . EF corta BC en I, y seconstruye exteriormente un triangulo equilatero BIJ . Encuentre ∠EJB.

39. (*) En la figura 60, se sabe que ∠AO1B−∠AO2B = 70◦ y ademas la tangente EB formael triangulo isosceles ABE, con AB = AE. Encuentre ∠EBC.

15M y K son distintos de A y B.

42

Page 43: Geometria Plana

Figura 60

40. (*) Dos circunferencias Γ1 y Γ2 se cortan en A y B. Una recta por A corta a Γ1 y Γ2 en Cy D, respectivamente, y la paralela a CD por B corta Γ1 y Γ2 en E y F , respectivamente.Demuestre que 4CDB ≡ 4EAF .

41. (*) La Recta de Simson-Wallace. Sean X, Y y Z los pies de las alturas trazadasdesde un punto P en el circuncırculo del 4ABC hacia AB, BC y CA, respectivamente.Demuestre que X, Y y Z estan alineados.

42. (*) Sea P un punto exterior al cuadrado ABCD tal que ∠APC = 90◦, Q es la interseccionde AB y PC, y R el pie de la perpendicular por Q a CA. Demuestre que P , R y D estanalineados.

43. En la figura 61, ABCD es un trapecio rectangulo tal que la circunferencia de diametroAB (y centro O) es tangente a CD. Demostrar que O pertenece a la circunferencia dediametro CD y que esta circunferencia es tangente a BA.

Figura 61

44. El 4ABC es rectangulo en C, la circunferencia de centro O es tangente a cada uno de loslados del 4ABC en los puntos P , Q y R (como se muestra en la figura 62), y se cumpleque AP = 20 y BP = 6. Calcule OP .

43

Page 44: Geometria Plana

Figura 62

45. En la figura 63 se muestran tres semicircunferencias, una de diametro AB (de centro Oy radio r), otra de diametro AO y la ultima de diametro OB. Determine la razon entreel radio de la circunferencia tangente a estas tres semicircunferencias y r.

Figura 63

46. El segmento AB es diametro de un semicırculo con centro en O. Un cırculo con centroen P es tangente a AB en O y tambien al semicırculo. Otro cırculo con centro en Q estangente a AB, al semicırculo y al cırculo de centro en P . Si AB = 2, ¿cual es el radiodel cırculo con centro en Q?

Figura 64

Problemas de Refuerzo.

47. Los vertices A y B de un triangulo equilatero 4ABC estan sobre una circunferencia deradio 1 y el vertice C esta en el interior de la circunferencia. Un punto D (distinto de B)que esta en la circunferencia es tal que AD = AB. La recta DC corta por segunda vez ala circunferencia en E. Encuentre la longitud del segmento CE. Ver figura 65.

48. (*) (OIM 2002, P-4) En un triangulo escaleno ABC se traza la bisectriz interior BD, conD sobre AC. Sean E y F puntos sobre la recta BD tales que (AE ‖ CF ) ⊥ BD, y seaM el punto sobre el lado BC tal que DM ⊥ BC. Demuestre que ∠EMD = ∠DMF .

44

Page 45: Geometria Plana

Figura 65

49. (*) (OMCC 2003, P-2) Sea S una circunferencia y AB un diametro de ella. Sea t la rectatangente a S en B y considere dos puntos C y D en t tales que B este entre C y D. SeanE y F las intersecciones de S con AC y AD y sean G y H las intersecciones de S conCF y DE. Demuestre que AH = AG.

50. (*) (The 59th Romanian Mathematical Olympiad District Round) Considere un cuadradoABCD y un punto E sobre el lado AB. La diagonal AC corta al segmento DE en el puntoP . La perpendicular por P a DE corta al lado BC en F . Probar que EF = AE + CF .

51. (*) Teorema de Arquımedes: En la figura 66, la region delimitada por tres semi-circunferencias mutuamente tangentes, es conocida como cuchilla de zapatero o arbelos.Demostrar que las circunferencias sombreadas son congruentes.

Figura 66: Teorema de Arquımedes.

45

Page 46: Geometria Plana

6. Teorema de Thales y su recıproco. Semejanza de Triangu-

los.

Introduccion.

Definicion

1. Razon: se llama razon, al cociente de dos cantidades, expresadas en la misma magnitud,por ejemplo a

b.

2. Proporcion: se llama proporcion a la igualdad de dos razones. Por ejemplo ab

= cd, 16 a los

terminos a y d se les llama extremos y los terminos b y c se les llama medios, al terminod se le llama cuarta proporcional entre a, b y c en este orden.

Propiedades de las proporciones:

1.a

b=c

dsi y solo si a· c = b· d.

2.a

b=c

dsi y solo si

b

a=d

coa

c=b

d.

3.a

b=c

dsi y solo si

a± bb

=c± dd

.

4.a

b=c

dsi y solo si

a+ b

a− b=c+ d

c− d.

Paralelismo y proporcionalidad.

Definicion

1. Un punto P ∈ AB divide al segmento AB en una razon dada r, si PAPB

= r.

Figura 67

2. Sean AB y CD dos segmentos, y sean P ∈ AB y Q ∈ CD, decimos que P y Q dividen aAB y CD en segementos proporcionales si PA

PB= QA

QB.

Figura 68

16En algunos textos de geometrıa se utiliza la notacion de proporcion ası a : b :: c : d que se lee “a es a b comoc es a d”.

46

Page 47: Geometria Plana

Teorema de Thales. Si tres paralelas cortan a dos secantes entonces los segmentos que de-terminan en ellas son proporcionales. 17

Antes de demostrar el Teorema de Thales, se enunciaran dos teoremas que a pesar de su apa-rente sencillez es de mucha utilidad en problemas que involucran Areas y Proporcionalidad.

Lema 1. Sea AB ‖ CD. Demuestre que: (ABC) = (ABD).

Lema 2. Sea P un punto sobre el lado AB (o su prolongacion) del 4ABC. Pruebe que:

AP

PB=

(APC)

(PBC)

.A continuacion se enuncian los pasos a seguir en la demostracion del teorema de Thales.Demostracion. Sean AA′, BB′ y CC ′ rectas paralelas que cortan a dos secantes en los puntosA, A′, B, B′, C, C ′ respectivamente (ver figura 69).

Figura 69: Teorema de Thales

Pruebe que:

1.AB

BC=

(ABB′)

(BCB′)

2.A′B′

B′C ′=

(A′B′B)

(B′C ′B).

3. (ABB′) = (A′B′B) y (BCB′) = (B′C ′B).

Con ayuda de las igualdades demostradas concluya que:

AB

BC=A′B′

B′C ′.

Observacion Importante: Utilice las propiedades de las proporciones para demostrar lasequivalencias siguientes (interpretelas geometricamente):

AB

BC=A′B′

B′C ′⇔ AC

AB=A′C ′

A′B′⇔ AC

BC=A′C ′

B′C ′

17El teorema de Thales puede enunciarse de manera general como sigue: Si tres o mas paralelas cortan a doso mas secantes entonces los segmentos que determinan en ellas son proporcionales.

47

Page 48: Geometria Plana

Corolario (Teorema de Thales en el triangulo). Toda recta paralela a un lado de un trianguloy que corte a los otros dos lados, divide a estos lados en segmentos proporcionales.

Recıproco del Teorema de Thales. Si tres rectas cortan a dos secantes en segmentos pro-porcionales y dos de estas rectas son paralelas entonces las tres rectas son paralelas.

Demostracion. Sean AA′, BB′ y CC ′ rectas que cortan a dos secantes en los puntos A, A′, B,

B′, C, C ′ respectivamente, tales que AA′ ‖ CC ′ yAB

BC=A′B′

B′C ′. Por el punto B tracemos una

recta paralela a AA′, la cual interseca a A′C ′ en el punto D (ver figura 70). Entonces, por el

Teorema de Thales se tiene que:AB

BC=A′D

DC ′. De donde,

A′B′

B′C ′=A′D

DC ′, ası por las propiedades

de las proporcionesA′C ′

B′C ′=A′C ′

DC ′, por lo que B′C ′ = B′D +DC ′ = DC ′ y por tanto B′D = 0,

o equivalentemente B′ = D y por lo tanto, BB′ ‖ AA′.

Figura 70: Recıproco del Teorema de Thales

Corolario (Recıproco del Teorema de Thales en el triangulo.) Si una recta interceptados lados de un triangulo en segmentos proporcionales entonces la recta es paralela al tercerlado del triangulo.

Triangulos semejantes. Decimos que el 4ABC es semejante al 4A′B′C ′ (Ver figura 71), locual denotamos ası ABC ∼ A′B′C ′, si:

AB

A′B′=

AC

A′C ′=

BC

B′C ′

y∠BAC = ∠B′A′C ′,∠ABC = ∠A′B′C ′,∠ACB = ∠A′C ′B′.

En los tres teoremas que se muestran a continuacion (los cuales son una consecuencia directa delTeorema de Thales) se establecen las condiciones mınimas para demostrar que dos triangulosson semejantes, a los cuales denominaremos: Criterios de Semejanza de Triangulos.

Primer criterio de semejanza de triangulos: Angulo-Angulo A-A. Si dos angulos deun triangulo son congruentes con dos angulos de otro triangulo, entonces los dos triangulos sonsemejantes.

48

Page 49: Geometria Plana

Figura 71: Definicion de Semajanza de Triangulos.

Demostracion. Supongamos que en el 4ABC y 4A′B′C ′ se tiene que ∠ABC = ∠A′B′C ′

y ∠ACB = ∠A′C ′B′, entonces ∠BAC = ∠B′A′C ′ (Por la suma de angulos internos en untriangulo).Sea D ∈ AB y E ∈ AC tales que AD = A′B′ y AE = A′C ′, dado que ∠DAE = ∠BAC =∠B′A′C ′, se sigue por L-A-L que 4ADE = 4A′B′C ′, por consiguiente ∠ADE = ∠A′B′C ′ =∠ABC, de donde DE ‖ BC (por ser iguales los angulos correspondientes) y por el teorema deThales

AB

AD=AC

AE

y por consiguienteAB

A′B′=

AC

A′C ′(1)

Sea F ∈ BC tal que DF ‖ AC, entonces FC = DE = B′C ′ (porque DECF es paralelogramoy por ser 4ADE = 4A′B′C ′) y por el teorema de Thales

BA

DA=BC

FC

o lo que es lo mismoAB

A′B′=

BC

B′C ′(2)

Luego, de (1) y (2) se tiene que:

AB

A′B′=

AC

A′C ′=

BC

B′C ′.

Ası, se ha demostrado que los tres pares de angulos son congruentes y los tres pares de ladosson proporcionales, por lo tanto, 4ABC ∼ 4A′B′C ′.

Segundo criterio de semejanza de triangulos: L-A-L. Si un angulo de un triangulo escongruente con otro angulo de otro triangulo y los lados que comprenden al angulo en el primertriangulo son respectivamente proporcionales a los lados que comprende al angulo en el segundotriangulo, entonces los dos triangulos son semejantes.

Demostracion. Suponga que el ∠BAC = ∠B′A′C ′ y queAB

A′B′=

AC

A′C ′. Considere los pun-

tos D y E, como en la demostracion del teorema anterior. Entonces por el criterio L-A-L,

49

Page 50: Geometria Plana

4ADE = 4A′B′C ′, de lo cual se deduce que ∠ADE = ∠A′B′C ′. Por otra parte tene-

mos que:AB

AD=

AC

AE, y al aplicar el recıproco del teorema de Thales, se puede afirmar que

DE ‖ BC, de lo cual a su vez se deduce que ∠ADE = ∠ABC, por angulos correspondientesentre paralelas. Finalmente por transitividad se concluye que ∠ABC = ∠A′B′C ′. Por lo tanto,4ABC ∼ 4A′B′C ′ (Por el criterio A-A.)

Tercer criterio de semejanza de triangulos: L-L-L. Si los tres lados de un triangulo sonrespectivamente proporcionales a los tres lados de otro triangulo, entonces los dos triangulosson semejantes.

Demostracion. Por hipotesis se tiene que:AB

A′B′=

AC

A′C ′=

BC

B′C ′y como antes sean D y E

puntos sobre AB y AC respectivamente tales que AD = A′B′ y AE = A′C ′. Entonces porel recıproco del teorema de Thales se tiene que DE ‖ BC y por consiguiente el ∠ABC =∠ADE y el ∠ACB = ∠AED, de donde 4ABC ∼ 4ADE (por el criterio A-A). Por endeAB

AD=

BC

DE, luego por transitividad

BC

DE=

BC

B′C ′, de donde DE = B′C ′. En consecuencia

4ADE = 4A′B′C ′ (por el criterio L-L-L), de lo cual se sigue que ∠A′B′C ′ = ∠ADE y∠A′C ′B′ = ∠AED, y por transitividad ∠A′B′C ′ = ∠ABC y ∠A′C ′B′ = ∠ACB =. Por lotanto, 4A′B′C ′ ∼ 4ABC (Por el criterio A-A.)

Ejercicios

1. Sean AB y CD las bases del trapecio ABCD, cuyas diagonales son perpendiculares. Sise sabe que AD = 13, AE = 12 y CE = 4 encuentre las longitudes de CD y AB.

2. En la figura 72, el 4ABC es equilatero, sus lados tienen longitud 3 y PA es paralela aBC. Si PQ = QR = RS, encontrar la longitud de CS.

Figura 72

3. Sea ABCD un trapecio de bases BC y AD, sus diagonales se cortan en E. Si BE = 3,ED = 4 y CE = 2, determine la medida de AE.

4. Las bases de un trapecio miden 3 y 5, y si su altura mide 4. Encontrar la distancia desdeel punto de corte de las diagonales hasta la base mayor.

50

Page 51: Geometria Plana

5. En la figura adjunta, el4ABC es rectangulo en A y el4ADBes rectangulo en D. El punto E es el punto de interseccion delos segmentos AD y BC. Si AC = 15, AD = 16 y BD = 12,calcule el area del 4ABE.

6. El4ABC es rectangulo en B. Se dibuja un rectangulo BEDF con D sobre la hipotenusa,E y F sobre BC y AB, respectivamente. Si AB = 1, demuestre que BC

BE= 1

1−DE .

7. Considerese los puntos A, B, C y D tales que A y B estan sobre el segmento OC yOD respectivamente, donde O es el centro de la circunferencia de radio r (Ver figura73). Si OA·OC = r2 = OB·OD, demuestre que el 4AOB ' 4DOC y que CD =(

r2

OA·OB

)AB.18

Figura 73

8. Sobre la circunferencia de centro O, se trazan los diametros AB y CD tales que AB ⊥ CD.Sea P un punto sobre el arco CBD y Q el punto de interseccion de las cuerdas AP yCD. Si DO = 1, demuestre que AP ·AQ = 2.

9. Un segmento de recta AB es divido por los puntos interiores K y L de manera queAL2 = AK·AB. Sea P un punto exterior al segmento AB tal que AP = AL. Pruebe que∠KPL = ∠LPB. Figura 74.

Figura 74

18La medida del segmento CD se denomina Distancia Inversa.

51

Page 52: Geometria Plana

10. En la figura 75, AB y AC son tangentes a la circunferencia, y CE ⊥ BD, siendo BD undiametro. Probar que BE·BO = AB·CE.

Figura 75

11. Demostrar que1

AX+

1

BY=

1

AZsi se cumple que AX ‖ BY ‖ CZ. (Ver figura 76.)

Figura 76

12. En la figura 77, el 4ABC es rectangulo. Se construyen exteriormente los cuadradosABEF y BCPQ. Demostrar que BM = BN .

Figura 77: .

52

Page 53: Geometria Plana

13. Sean O, P y R los centros de las tres circunferencias. Si OR = r y Q es la interseccionde PO con la circunferencia de centro R, demuestre que OP ·OQ = r2. Ver figura 78.

Figura 78

14. Si en un triangulo rectangulo se traza la altura correspondiente a la hipotenusa, entonces:

a) Los dos nuevos triangulos que resultan, son semejantes entre si y semejantes altriangulo original.

b) La altura es media proporcional 19 entre los segmentos que ella determina sobre lahipotenusa.

c) Cada cateto es media proporcional entre la hipotenusa y la proyeccion del catetosobre la hipotenusa.

d) Demuestre el teorema de Pitagoras.

15. Si dos triangulos tienen sus lados respectivamente paralelos o respectivamente perpendi-culares, entonces los dos triangulos son semejantes.

16. Las alturas, las bisectrices y las medianas homologas de dos triangulos semejantes estanen la misma razon que sus lados homologos.

17. Sean ABC y A′B′C ′ dos triangulos semejantes con ABA′B′ = BC

B′C′ = CAC′A′ = k. Demuestre

que: la razon entre los perımetros de los triangulos es k y que la razon entre sus areas esk2.

18. Teorema de Menelao. Dado el 4ABC, sea P un punto sobre la recta AB, Q un puntosobre la recta BC, R un punto sobre la recta CA. Si los puntos P , Q, R estan alineados

entoncesAP

PB

BQ

QC

CR

RA= 1.

Para demostrar este teorema, sea W un punto sobre la recta PQR tal que BW ‖ AC:

a) Demuestre que los triangulos APR y BPW son semejantes.

19Si b es una magnitud tal que ab = b

c , entonces decimos que b es media proporcional entre a y c,o de maneraequivalente: b es media proporcional entre a y c si y solo si b2 = a· c.

53

Page 54: Geometria Plana

Figura 79: Teorema de Menelao.

b) Demuestre que los triangulos CQR y BQW son semejantes.

c) De los literales a) y b) deduzca queAP

PB

BQ

QC

CR

RA= 1.

19. Teorema de Ceva. Dado el4ABC, sea P un punto sobre elrecta AB, Q un punto sobre la recta BC y R un punto sobrela recta CA. Si las rectas AQ, CP , BR concurren, entoncesAP

PB

BQ

QC

CR

RA= 1.

Para demostrar este teorema, sean W y V los puntos de interseccion de la recta que pasapor B paralela a AC, con las rectas CP y AQ, respectivamente.

a) Demuestre que 4APC ∼ 4BPW y que 4AQC ∼ 4V QB.

b) Demuestre que 4BWP ∼ 4RCP y que 4BV P ∼ 4RAP .

c) Utilice los literales a) y b) para probar queAP

PB

BQ

QC

CR

RA= 1.

20. Si dos cuerdas se interceptan en el interior de una circunferencia entonces el productode las medidas de los segmentos determinados por el punto de interseccion en una de lascuerdas es igual al producto de las medidas de los segmentos determinados en la otracuerda.

21. Si dos segmentos se interceptan en un punto que esta en el interior de los dos segmentosy el producto de las medidas de los segmentos determinados por el punto de interseccionen el primer segmento es igual al producto de las medidas de los segmentos determinadospor el punto en el segundo segmento,entonces los extremos de los segmentos estan sobreuna circunferencia.

22. Si desde un punto P exterior a una circunferencia se trazan dos semirrectas secantesque cortan a la circunferencia en los puntos A, B y C, D respectivamente, entoncesPA·PB = PC·PD.

54

Page 55: Geometria Plana

23. Si desde un punto P se trazan dos semirrectas con los puntos A, B sobre una y los puntosC, D sobre la otra, tales que PA·PB = PC·PD, entonces los puntos A, B, C, D estansobre una circunferencia.

24. Si desde un punto exterior a una circunferencia se trazan dos semirrectas, una tangentey la otra secante, entonces el segmento entre el punto y el punto de tangencia es mediaproporcional entre los segmentos determinados entre el punto exterior y los puntos deinterseccion de la secante con la circunferencia. 20

25. Si P es un punto sobre el mismo plano que una circunferencia de centro O y radio r, y des la distancia del punto P al centro O de la circunferencia, demuestre que:

a) Si P esta en el interior de la circunferencia, entonces la potencia de P es r2 − d2.

b) Si P esta en el exterior de la circunferencia, entonces la potencia de P es d2 − r2.

c) Si P esta sobre de la circunferencia, entonces la potencia de P es cero.

26. (*) (IV OMCC, P-4) Sea ABC un triangulo, D el punto medio de BC, E un punto sobreel segmento AC tal que BE = 2AD y F el punto de interseccion de AD con BE. Si∠CAD = 60°, encuentre la medida de los angulos del 4FEA.

27. (*) Sea ABCD es un trapecio con AD ‖ BC. M y N son los puntos medios de CD y BC,respectivamente, y P el punto comun de las rectas AM y DN . Si PM

AP= 1

4, demuestre que

ABCD es paralelogramo.

28. Dado el 4ABC se construye un cuadrado PQRS con P en AB, Q en AC, R y S en BC.Sea H el pie de la altura desde A hacia BC. Demuestre que:

a)1

PQ=

1

AH+

1

BC

b) (ABC) = 2(PQRS) si y solo si AH = BC.

29. Sea P un punto en el interior del 4ABC. Se trazan por P las paralelas a los lados deltriangulo, que queda dividido en tres triangulos y tres paralelogramos. Si las areas de lostres triangulos de la subdivision son, en algun orden, 9, 16 y 25, hallar el area del 4ABC.

Problemas de Refuerzo.

30. (*) En la figura anexa, BC = CD = DE =EA = x y ∠AEB = 90°. Demuestre que∠ABC + ∠ACD + ∠ADE = 90°.

20Los problemas anteriores nos permite establecer la siguiente definicion de Potencia de un punto conrespecto a una circunferencia: La potencia de un punto P con respecto a una circunferencia de centro O yradio r es el producto PA·PB, donde A y B son los puntos de interseccion de la circunferencia con una rectaque pasa por P .

55

Page 56: Geometria Plana

31. Las tres circunferencias de la figura 80 tienen el mismo radio r, sus centros son colinealesy la circunferencia de centro O2 es tangente a las otras dos. Por A se traza una tangentea la circunferencia de centro O3. Obtenga el valor del segmento BC en funcion de r.

Figura 80

32. Sea ABCD un rombo, con AC = 6 y BD = 8. Se construyen exteriormente los cuadradosADEF y CDHG, cuyos centros son O1 y O2, respectivamente (Vea figura 81). Calcularla medida del segmento O1O2.

Figura 81

33. Dado un paralelogramo ABCD, se trazan dos circunferencias tangentes externamenteen P , y tales que la primera es tangente internamente al ∠ABC y la otra es tangenteinternamente al ∠CDA, como en la figura 82. Demuestre que B, P y D estan alineados.

34. Sea ABCD un cuadrado con P y Q sobre AB y BC tales que BP = BQ. Sea H el piede la perpendicular de B a PC. Demuestre que DHQ = 90°.

56

Page 57: Geometria Plana

Figura 82

35. En un 4ABC el ∠CAB = 120. Encuentre la medida de la bisectriz interna del ∠CABen funcion de los lados adyacentes.

36. El 4ABC tiene lados de 13, 14 y 15 unidades. El 4A′B′C ′ esta dentro del 4ABC conlados paralelos a los de este y a 2 unidades de distancia de los lados del mismo. Calcule(ABC)− (A′B′C ′).

37. (*) (Asiatico Pacıfica) Sea ABC un triangulo y D el pie de la altura con respecto a A.Sean E y F puntos en una recta que pasa por D (distintos de D) tales que AE ⊥ CEy AF ⊥ BF . Sean M y N los puntos medios de BC y EF , respectivamente. Demuestreque AN ⊥ NM .

57

Page 58: Geometria Plana

7. Puntos y Rectas Notables del Triangulo.

MEDIANAS

Definicion: En un triangulo, una mediana es el segmento de recta que une un vertice con elpunto medio del lado opuesto.

Teorema: Las tres medianas de un triangulo concurren en un punto llamado el Centroide21

del triangulo y usualmente es denotado por G. Ademas, las medianas de cortan mutuamenteen razon 2:1.

Demostracion: Dado el 4ABC sean A′, B′, C ′, los puntos medios de BC, CA, AB, respec-tivamente. Defina G como la interseccion de BB′ con CC ′. Por el teorema de la base media,B′C ′ ‖ BC y 2B′C ′ = BC; observe que 4BCG ' B′C ′G, con razon de semejanza 2, por loque

GB

GB′=GC

GC ′= 2

Analogamente, si G∗ = AA′ ∩BB′ se cumple

G∗B

G∗B′=G∗A

G∗A′= 2

Ası, G y G∗ dividen al segmento BB′ en dos segmentos cuya razon es 2:1, por lo que G = G∗,lo cual implica que AA′, BB′, CC ′ concurren y

GA

GA′=GB

GB′=GC

GC ′= 2

MEDIATRICES

Definicion: Dado un segmento AB, la mediatriz del segmento es el lugar geometrico de puntosque equidistan de A y B, i.e., un punto P esta sobre la mediatriz de AB si y solo si PA = PB.

Teorema: La mediatriz de AB es una recta l perpendicular a AB y que pasa por su puntomedio.

Demostracion: Sea M el punto medio de AB, y l pasa por M y l ⊥ AB. En primer lugarse probara que todos los puntos de l satisfacen la definicion de mediatriz: Por definicion depunto medio MA = MB. por lo que claramente M pertenece a la mediatriz de AB; sea P unpunto de l distinto de M , por criterio LAL, 4PMA ≡ 4PMB por lo que PA = PB. Ahora,cabe preguntarse si existe algun punto fuera de l que tambien cumpla la definicion: supongaP ′ tal que P ′A = P ′B, esto implica que 4P ′AB es isosceles, y entonces ∠P ′AB = ∠P ′BA;si M ′ es la proyeccion de P ′ sobre AB, por criterio ALA 4P ′AM ′ ≡ 4P ′BM ′, lo cual impli-ca que M ′A = M ′B, es decir que M ′ = M , y esto obliga a que P ′ este sobre l (ya que P ′M ′ = l).

Teorema: Las mediatrices de un 4ABC concurren en un punto que equidista de los verticesdel triangulo, llamado el Circuncentro del 4ABC

21Tambien conocido como Geocentro, Centro de Gravedad, Baricentro, o mas formalmente Equibaricentro.

58

Page 59: Geometria Plana

Figura 83: Concurrencia de Mediatrices, Circuncentro y Circuncırculo.

Usualmente, el circuncentro es denotado por O, y R representa la distancia del circuncentro alos vertices

R = OA = OB = OC

A esta distancia se le llama Circunradio del 4ABC. Ası, O es el centro de una circunferenciaque pasa por A, B, C, cuyo radio es R, llamada el Circuncırculo del 4ABC.22

Demostracion: Sea O la interseccion de las mediatrices de AB y BC, por el teorema anterior,como O pertenece a la mediatriz de AB se cumple OA = OB, y como tambien pertenece a lamediatriz de BC, OB = OC; entonces OC = OA, y utilizando de nuevo el teorema anterior,O debe pertenecer a la mediatriz de CA. Ası, las tres mediatrices concurren en O, y este puntoequidista de los vertices del 4ABC.

Corolario: Dado un triangulo, existe una circunferencia que pasa por los tres vertices (el cir-cuncırculo); ademas, esta circunferencia es unica.

Una observacion importante es que la mediatriz del lado de un triangulo NO siempre pasa porel vertice opuesto; de hecho, esto solo se da si el triangulo es isosceles.

ALTURAS

La altura es un concepto que esta intrınsecamente relacionado con la distancia de un punto auna recta; la altura es la recta que debe trazarse para determinar esta distancia, i.e., es unarecta que pasa por el punto y es perpendicular a la recta. A la interseccion entre la altura y larecta generalmente se le llama pie de la altura, o tambien (mas formal) proyeccion del puntosobre la recta. En particular, para triangulos, definiremos la altura de la siguiente forma:

Definicion: Dado un triangulo, una altura es una recta que pasa por un vertice y es perpen-dicular al lado opuesto.

22En ocasiones, denotaremos a esta circunferencia por Γ(ABC).

59

Page 60: Geometria Plana

Es importante observar que el pie de la altura NO siempre pertenece a un lado; de hecho, unaaltura puede estar “al interior” de un triangulo, coincidir con un lado, o estar completamenteafuera de un triangulo.

Teorema: Las alturas de un triangulo concurren en un punto, llamado el Ortocentro del triangu-lo, usualmente denotado por H.23

Demostracion: Dado el 4ABC, se construyen los puntos A1, B1, C1, tales que ABA1C,BCB1A, CAC1B son paralelogramos. Observe que el4ABC es el triangulo medial del4A1B1C1,y que las alturas del 4ABC son las mediatrices del 4A1B1C1; como las mediatrices de untriangulo concurren (en este caso, las del 4A1B1C1), las alturas del 4ABC concurren.

La altura tambien puede escribirse en terminos de lugar geometrico:

Teorema: La recta l es perpendicular a AB si y solo si AL2−LB2 es constante. Es decir, queuna recta perpendicular a AB es el lugar geometrico de los puntos L que satisfacen la condicionanterior.

Demostracion: sea P la interseccion de l con AB, y L un punto arbitrario sobre l; por Pitago-ras se tiene AL2 − LB2 = AP 2 − PB2, y el termino derecho de la igualdad es constante. Laotra direccion de la implicacion se prueba por contradiccion.

De esa definicion tambien puede fabricarsele una demostracion del teorema anterior, sin em-bargo, no se aborda porque la prueba se basa en un resultado sofisticado llamadado Teoremade Steiner.24

BISECTRICES

Definicion: La bisectriz de un angulo es una recta que “divide” al angulo en dos angulos deigual magnitud.

Teorema: El lugar geometrico de puntos que equidistan de dos rectas dadas, generan un parde rectas perpendiculares llamadas bisectriz interna y bisectriz externa del angulo formado porlas rectas.

Demostracion: Suponga que las rectas se cortan en un punto O; sean a, b las rectas dadas, yP un punto que equidista de ellas; si A y B son las proyecciones de P sobre a y b, respectiva-mente, entonces PA = PB. Observe que por criterio LLL (utilizando Pitagoras previamente),4OAP ≡ 4OBP , por lo que ∠POA = ∠POB, i.e., P pertenece a la bisectriz del ∠AOB.Claramente aquı se dan dos casos, recuerde que para definir el angulo entre a y b se utilizan uni-camente semi-rectas, por lo que las rectas a y b definen cuatro angulos, que por parejas pueden

23El triangulo formado por los pies de las alturas de un 4ABC es llamado el triangulo ortico del 4ABC.24Sean l, m, n, tres rectas perpendiculares a los lados del AB, BC, CA del 4ABC, respectivamente. Sean

L, M , N , puntos arbitrarios sobre l, m, n, respectivamente. Entonces las rectas l, m, n concurren si y solo siAL2 + BM2 + CN2 = NA2 + LB2 + MC2.

60

Page 61: Geometria Plana

ser opuestos por el vertice o suplementarios; de estos se escoge cualquiera de ellos como referen-cia, entonces, si ∠AOB coincide con este o con el opuesto por el vertice, la recta PO es llamadabisectriz interna, y en caso contrario, bisectriz externa. Ası, el lugar geometrico son dos rectas,y su perpendicularidad se basa en los pares de angulos que son suplementarios. Finalmente, sia ‖ b, el lugar geometrico es una recta paralela a a y b que se ubica entre ellas a igual distancia deambas (este es un caso extrano de bisectriz interna, sin embargo, en ocasiones es util tener estaconvencion en mente; peor aun, la bisectriz externa es una recta ideal llamada recta al infinito).

Teorema: Las bisectrices internas de un 4ABC concurren en un punto, llamado el Incentrodel 4ABC, usualmente denotado por I. La distancia de I a los tres lados del triangulo es iguala un numero r, llamado el Inradio del 4ABC, y de aquı que la circunferencia de centro I yradio r sea tangente a los lados del triangulo; dicha circunferencia es llamada el Incırculo del4ABC.25

Figura 84: Concurrencia de Bisectrices Internas, Incentro e Incırculo.

Demostracion Sea I la interseccion de las bisectrices internas de ∠A y ∠B (obviamente, Iesta en el interior del 4ABC); como I pertenece a la bisectriz interna del ∠A, por el teoremaanterior dist(I, AB) = dist(I, AC), y analogamente, como I pertenece a la bisectriz interna del∠B, dist(I, AB) = dist(I, CB); entonces dist(I, AC) = dist(I, CB), y de nuevo por el teoremaanterior y dado que I esta al interior del triangulo, I pertenece a la bisectriz interna del ∠C.Ası, las tres bisectrices internas concurren en un punto que equidista de los lados del triangulo.

Es importante notar que las interseccion de una bisectrices interna con el lado opueto deltriangulo NO siempre coincide con el puntos de tangencia del incırculo;26 de hecho, esto ocurresolamente si el triangulo es isosceles.

Corolario: Dado un triangulo, existe una circunferencia que es tangente interiormente a lostres lados (el incırculo); ademas, esta circunferencia es unica.27

25En algunas ocasiones denotaremos al incırculo por Λ(ABC).26En la figura, el 4ABC es llamado triangulo tangencial del 4DEF .27Existen 3 circunferencias mas que son tangentes a los tres lados del triangulo, llamados excırculos; estas

circunferencias se ubican en el exterior del triangulo.

61

Page 62: Geometria Plana

Ejercicios

1. Las areas de los seis triangulos AGB′, AGC ′, BGA′, BGC ′, CGA′, CGB′ son iguales eiguales a un 1

6del area del triangulo ABC.

Figura 85

2. Los cuatro triangulos AB′C ′, BC ′A′, CA′B′, A′B′C ′,28 son congruentes entre si y seme-jantes al 4ABC con razon de semejanza 1

2.

3. El centroide del 4ABC coincide con el centroide del triangulo medial 4A′B′C ′. Ademas,estos dos triangulos tienen lados correspondientes paralelos (triangulos homoteticos).

4. En la figura 86, G es el centroide. Si GD = 2 y el area sombreada vale 5, calcule AD y el(ABC).

Figura 86

5. Demostrar que las paralelas a los lados de un 4ABC, trazadas por el centroide G dividencada lado en tres partes iguales.

6. ABCD es un paralelogramo de centroide (baricentro) E, M es el punto medio de AD,y F es la interseccion de AC con BM . Si el area de ABCD es 1, calcule el area delcuadrilatero DEFM .

7. En el 4ABC, se traza la mediana AM . Demostrar que si BM = AM , entonces eltriangulo es rectangulo en A.

28El 4A′B′C ′ es llamado el triangulo medial del 4ABC.

62

Page 63: Geometria Plana

8. La suma de las distancias del centroide a los puntos medios de los lados de un trianguloes 20. Calcule la suma de las medianas del triangulo.

9. La mediana tiene longitud menor que la semisuma de los lados adyacentes, es decir AA′ <b+ c, BB′ < c+ a, CC ′ < a+ b.

10. Dado el 4ABC, sean D y E puntos variables sobre los lados AB y AC respectivamentetales que BC ‖ DE. Entonces, la mediana AA′ puede definirse como el lugar geometricode los puntos P tales que P ∈ CD ∩BE.29

11. Siempre es posible construir un triangulo XY Z con las medianas AA′, BB′, CC ′ de un4ABC dado. Ademas, los segmentos que unen el centroide del 4XY Z con sus verticesson iguales a la mitad de los lados del 4ABC.

12. En el 4ABC, AB = BC y la mediatriz de BC interseca a la mediana BM en L. Si∠LCB = 25, determine la medida del ∠LAC.

13. Ley del Seno. Dado un 4ABC, se cumple que

sen∠Aa

=sen∠B

b=sen∠C

c=

1

2R

14. Las reflexiones de H con respecto a los lados del 4ABC caen sobre el circuncırculo delmismo, es decir HHa = HaX y analogo para los otros lados.

Figura 87

15. Las reflexiones de H con respecto a los puntos medios de los lados del triangulo, caensobre el circuncırculo del mismo.

29Si D = A se define P = A, y cuando D = B entonces P es punto medio de BC.

63

Page 64: Geometria Plana

16. Si O y H son el circuncentro y el ortocentro de un 4ABC, respectivamente, entonces∠BAH = ∠CAO.

17. La altura AHa es bisectriz del ∠HbHaHc.

18. Los circuncırculos de 4ABC, 4ABH, 4BCH, 4CAH tienen igual radio.

19. La perpendicular trazada desde A al lado HbHc del triangulo ortico, pasa por el circun-centro del 4ABC.

20. A, B, C y H forman un cuadrilatero ortocentrico, es decir que cada punto es el ortocentrodel triangulo formado por los otros tres.

21. El ortocentro de un triangulo esta al interior, sobre un vertice, o afuera del triangulo, siel triangulo es acutangulo, rectangulo, u obtusangulo, respectivamente.

22. El circuncentro del 4ABC es el ortocentro del triangulo medial 4A′B′C ′.

23. Sea O el circuncentro del 4ABC. Si ∠AOC = 100 y ∠OCB = 30, determine la medidade los angulos del 4ABC.

24. Hallar los angulos de un triangulo cuyo triangulo ortico tiene angulos de 20, 50 y 110.

25. Sea ABC un triangulo obtusangulo de circuncentro O y altura AD. Si ∠OAB = 25 y∠OCB = 15, calcule el ∠DAB.

26. El 4ABC de circuncentro O y altura BD. Si ∠DAB = 35 y ∠OBD = 10 encontrar losangulos del triangulo ABC.

27. En la figura 88, AB es diametro de la circunferencia. Si X es la interseccion de CG conAB, calcular el ∠CXB.

Figura 88

64

Page 65: Geometria Plana

28. En el 4ABC, se trazan la altura AH y la mediana BM . Demuestre que el 4MHC esisosceles.

29. Un 4ABC es rectangulo en C, ∠A = 75 y CH es altura. Demuestre que CH =AB

4.

30. Sea O el circuncentro del 4ABC con ∠C = 45 y sea D el pie de la altura desde A.Calcule la medida del ∠ODC.

31. Dado el 4ABC isosceles con ∠A = 90, sean P y Q son puntos dentro del triangulo talesque BP = AQ y AP = CQ. Si BP y CQ se cortan en R, demostrar que AR ⊥ PQ.

32. Se ubican los puntos M y K sobre los lados BC y CD del cuadrado ABCD, respecti-vamente, de modo que MC = KD. Sea P la interseccion de MD y BK, demuestre queAP ⊥MK.

33. Sean D, E, F los puntos de tangencia del incırculo sobre los lados BC, CA, AB del4ABC. Demuestre que se cumplen las siguientes relaciones, donde s denota el semi-perımetro del triangulo:

AE = AF = s− aBD = BF = s− bCD = CE = s− c

34. El ortocentro del 4ABC es el incentro de su triangulo ortico.

35. Dado un 4ABC, su triangulo ortico y su triangulo tangencial tienen lados correspon-dientes paralelos (triangulos homoteticos).

36. Las bisectrices exteriores de ∠B y ∠C, junto con la bisectriz interior de ∠A, concurrenen un punto, llamado el Excentro con respecto al vertice A, usualmente denotado por Ia.Este punto es equidistante a los lados del 4ABC, dicha distancia es el Exradio respecto aA, usualmente denotado por ra. Ası, la circunferencia de centro Ia y radio ra es tangenteexteriormente a los lados del 4ABC, y es llamada el Excırculo respecto a A.30

37. I es ortocentro del 4IaIbIc. Ademas se cumple:

AX = AZ = sBX = BY = s− bCY = CZ = s− c

38. En un 4ABC, la bisectriz exterior del ∠ABC y la bisectriz exterior del ∠BCA se cortanen D. La paralela a BC por D corta a AC en L y a AB en M . Si LC = 5 y MB = 7,hallar LM .

39. El 4ABC es rectangulo en A. Si I es el incentro, calcular ∠BIC.

40. En un 4ABC, el ∠ABC − ∠CAB = 90. Sean D y E los pies de las bisectrices interiory exterior del ∠BCA respectivamente. Demuestre que CD = CE.

30Analogamente se definene los excırculos con respecto a los otros vertices.

65

Page 66: Geometria Plana

Figura 89

41. En el4ABC, AB < AC, AD es bisectriz, y E es un punto en AB tal que el ∠EDB = 90.El punto F sobre AC es tal que el ∠BED = ∠DEF . Demuestre que el ∠BAD = ∠FDC.

42. En el 4ABC se trazan las bisectrices interiores BD y CE tales que D es el punto sobreAC, E es el punto sobre AB, 2∠BDE = 3∠B y ∠CED = 2∠B. Calcular los angulos del4ABC.

43. Dado el 4ABC con ∠A = 90, sea D el pie de la perpendicular desde A. Sean ademas Iy J los incentros respectivos de 4ABD y 4ACD. Demostrar que la bisectriz del ∠BACes perpendicular a IJ .

44. Un triangulo es isosceles si cumple alguna de las siguientes condiciones:

a) Dos medianas son iguales.

b) Dos alturas son iguales.

c) Dos bisectrices son iguales.31

45. Teorema de la Bisectriz: Dado el 4ABC, sean P y P ′ sobre BC. Se cumple que APy AP ′ son la bisectriz interna y la bisectriz externa del ∠A si y solo si

BP

PC=BA

AC=BP ′

P ′C

Sugerencia: Para demostrar la primera igualdad, trace CD ‖ AP con E sobre la prolon-gacion de AB.

31Este caso es aparentemente tan sencillo como los anteriores, pero realmente es un resultado muy complicadoy recibe el nombre de Teorema de Steiner-Lehmus.

66

Page 67: Geometria Plana

46. (*) De acuerdo con los datos de la grafica 90, calcular el valor de AB.

Figura 90

47. Dos circunferencias son tangentes internamente en P , y una cuerda AB de la circunfe-rencia de radio mayor es tangente en Q a la otra circunferencia. Ver figura 91.

a) Demuestre que PQ es bisectriz del ∠APB.

b) Llame A′ y B′ a las otras intersecciones de PA y PB con la circunferencia de radiomenor y suponga que AB = 15, PA′ = 3 y PB′ = 2; calcule AQ y BQ.

Figura 91

48. (*) Sea ABC un triangulo tal que las medianas respectivas a B y C son perpendiculares.Demuestre que se cumple la relacion.

5BC2 = CA2 + AB2.

49. (*) Teorema de Poncelet: Demuestre si 4ABC es un triangulo rectangulo con ∠A =90°, entonces 2(r +R) = b+ c.

67

Page 68: Geometria Plana

Problemas de Refuerzo.

50. (*) Sea ABCD un paralelogramo. Q es el punto medio de AD,F el pie de la perpendicularpor B sobre QC. Probar que AF = AB.

51. Dado el rombo ABCD, se trazan las bisectrices internas de ∠DAC, ∠CAB, ∠BCA,∠ACD, y cortan a DC, CB, BA, AD en P , Q, R, S, respectivamente. Demuestre quePQRS es un rectangulo.

52. (*) Sea ABCD un cuadrilatero tal que AB = CD. Las mediatrices de AC y BD se cortanen P . Probar que ∠PAC = ∠PCA = ∠PBD = ∠PDB.

53. (*) ABC es un triangulo y P un punto en su interior. Sean A′, B′ y C ′ las reflexiones deP sobre BC, CA y AB, respectivamente. D, E y F son los pies de las perpendicularesrespectivos desde A, B y C hacia B′C ′, C ′A′ y A′B′. Probar que AD, BE y CF sonconcurrentes.

54. (*) (Arnoldo Aguilar) En la figura 92, ABGH, BCFG y CDEF son cuadrados. Si I esel centro de ABGH y J = DH ∩BG, demuestre que I, J y F estan alineados.

Figura 92

55. (*) (Arnoldo Aguilar) Sea ABC un triangulo equilatero. M y N son los puntos medios deAB y BC, respectivamente. Exteriormente al4ABC se construye un triangulo rectanguloisosceles 4APC, con ∠APC = 90◦. Si I es la interseccion de AN y MP , demuestre queCI es la bisectriz de ∠ACM .

56. (*) En la figura adjunta, el 4ABC es tal que ∠A = 90° y∠B = 60°. ¿Cual es el radio de la circunferencia?

57. (*) Dado el paralelogramo ABCD, sea M el punto medio de AB, y N la interseccion deCD con la bisectriz interna del ∠ABC. Demuestre que MC ⊥ BN si y solo si AN esbisectriz del ∠DAB.

68

Page 69: Geometria Plana

58. (*) En el 4ABC, se sabe que los vertices B, C, el circuncentro O y el ortocentro H del4ABC estan todos sobre una misma circunferencia.

a) Calcule el valor de ∠A.

b) Demuestre que el incentro tambien pertenece al circuncırculo de BCOH.

59. (*) Sea ABC un triangulo de ortocentro H. Sean P y Q los pies de las perpendicularesdesde H a las bisectrices interior y exterior de A, respectivamente. Si M es el punto mediode BC, mostrar que P , Q y M estan alineados.

60. (*) En un triangulo ABC, sea M el punto medio de BC. Si se cumple que AB 6= AC yademas ∠MAC + ∠ABC = 90◦, hallar ∠BAC.

61. (*) Sea ABC un triangulo y U un punto de su circuncırculo tal que AU es bisectriz. Lasmediatrices en AB y AC cortan a AU en X y Y . Sea T la interseccion de BX con CY .Demostrar que AU = TB + TC.

62. (*) (The 59th Romanian Mathematical Olympiad Final Round) Sea ABCD un rectangulode centro O con AB 6= BC. La perpendicular en O a BD corta a las lıneas AB y BC enlos puntos E y F , respectivamente. Sean M y N los puntos medios de los segmentos CDy DA, respectivamente. Probar que las lıneas rectas FM ⊥ EN .

63. (*) Sea ABC un triangulo rectangulo, con A = 90◦. Sea D un punto en su interior talque ∠DAC = ∠DCA = ∠DBC = α, y AC = BD. Determine el valor de α.

64. (*) Sea ABC un triangulo y M un punto tal que ∠MAB = 10, ∠MBA = 20, ∠MAC =40 y ∠MCA = 30. Probar que el 4ABC es isosceles.

65. (*) En la figura 93, ABCD y PQRS son cuadrados, 4ABP ≡ 4BCQ ≡ 4CDR ≡4DAS y los los radios de las cinco circunferencias son iguales a r. Si a es el lado delcuadrado ABCD, determine r en funcion de a.

Figura 93

69

Page 70: Geometria Plana

66. (*) Recta de Euler. El centroide G, el ortocentro H y el circuncentro O de un trianguloestan alineados, y ademas GH = 2GO.

67. Circunferencia de los 9 puntos:32 Dado un 4ABC de ortocentro H, se cumple quelos puntos medios de los lados, los pies de las alturas, y los puntos medios de HA, HB,HC, se ubican sobre una misma circunferencia. Ademas, el centro de esta circunferenciaes el punto medio de HO y su radio es R

2, donde O y R son el circuncentro y el circunradio

del triangulo.Para demostrar este resultado se sugiere seguir los siguientes pasos:

a) Si Ha es el pie de la altura trazada desde A, demuestre que la reflexion de H conrespecto a Ha pertenece a circuncırculo del 4ABC. Resultados similares se cumplenpara Hb y Hc.

b) Si A′ es el punto medio de BC, demuestre que la reflexion de H con respecto a A′

pertenece al circuncırculo del 4ABC.

c) De los resultados anteriores, observe que hay 9 puntos sobre el circuncırculo del4ABC: los vertices, las reflexiones de H con respecto a los pies de las alturas, ylas reflexiones de H con respecto a los puntos medios de los lados; a partir de esto,concluya que los puntos medios de los segmentos que van de H a estos 9 puntos,tambien deben pertenecer en una misma circunferencia.

d) Concluya ademas que el centro de esta nueva circunferencia es el punto medio de HO.

Otro camino de solucion es el siguiente:

a) Sea 4A′B′C ′ el triangulo medial del 4ABC. Pruebe que ∠A′B′C ′ = ∠BHaC′ y

concluya que HaA′B′C ′ es un cuadrilatero cıclico; los mismo debe cumplirse para Hb

y Hc.

b) Sea X el punto medio de HA. Demuestre que ∠B′A′C ′+∠B′XC ′ = 180 y concluya queXC ′B′A′ es un cuadrilatero cıclico; lo mismo debe cumplirse para los puntos mediosde HB y HC.

c) De lo anterior, concluya que los pies de las alturas y los puntos medios de los segmentosque van desde H hasta los vertices del 4ABC, se ubican sobre el circuncırculo del4A′B′C ′.

d) Si N es el circuncentro del 4A′B′C ′, demuestre que N , O, G forman la recta de Eulerdel 4A′B′C ′ y utilice sus propiedades para probar que N es el punto medio de HO.

68. El area del 4ABC, denotada por [ABC], cumple:

[ABC] =base× altura

2

[ABC] =ab sen∠C

2=bc sen∠A

2=ca sen∠B

2=abc

4R[ABC] = sr

[ABC] =√s(s− a)(s− b)(s− c). (Formula de Heron).

32Tambien conocida como Circunferencia de Feuerbach.

70

Page 71: Geometria Plana

69. El circunradio, el inradio y los exradios de un triangulo cumplen:

4R = ra + rb + rc − r[ABC] = ra(s− a) = rb(s− b) = rc(s− c)

r =

√(s− a)(s− b)(s− c)

s

ra =

√s(s− b)(s− c)

s− a

rb =

√s(s− a)(s− c)

s− b

rc =

√s(s− a)(s− b)

(s− c)

70. Dado el 4ABC, sea I el incentro e Ia el excentro respecto a A.

a) Demuestre que BICIa es un cuadrilatero cıclico.

b) Si M es la interseccion de IIa con el circuncırculo del triangulo (M 6= A), demuestreque dicho punto es el circuncentro de BICIa.

c) Sea M ′ el punto diametralmente opuesto a M en el circuncırculo, y sea P la proyeccionde I sobre AB. Demuestre que 4M ′CM ' 4AIP .

d) Sea O el circuncentro del triangulo Calcule la potencia de punto de I con respectoal circuncırculo, y utilizando los resultados anteriores deduzca la Formula de Euler :OI2 = R2 − 2Rr.

e) A partir de la formula de Euler demuestre que R ≥ 2r.

71

Page 72: Geometria Plana

8. Solucion a Problemas Selectos.

Teoremas Fundamentales del Triangulo.

1. En la figura 94, ABDE es un cuadrado y BCD es un triangulo isosceles con BD = DC.Si ∠ABC = 160, determinar la medida de ∠AEC.

Figura 94

Solucion: ∠DBC = ∠DCB = 160−∠ABD = 70, de donde se obtiene que ∠BDC = 40y ∠EDC = ∠EDB + ∠BDC = 130. Como 4EDC es isosceles, entonces ∠DEC =∠DCE = 25. Por lo tanto ∠AEC = 90− ∠DEC = 65.

2. Hallar la suma de los angulos α + ε+ θ + φ en la figura 95.

Figura 95

Solucion: ∠CAB = θ, ∠EDC = α por ser opuestos por el vertice. Como el ∠AFDexterno en el 4BDF , se tiene ∠AFD = α+ ε. Sumando los angulos internos del 4AEFse tiene α + φ+ θ + ε = 180°.

3. (XV Competencia de Clubes Cabri Primera Ronda) En la figura 96, ABCD es unrectangulo tal que AB = 2BC. M es el punto medio de AB y los triangulos AMEy MBF son equilateros. Si P es la interseccion de las rectas DE y CF , encuentre losangulos del 4CDP .

72

Page 73: Geometria Plana

Figura 96

Solucion: Note que AD = AE = FB = BC por lo que 4DAE y 4BCF son ambosisosceles. Luego ∠DAE = ∠CBF = 90+60 = 150 lo que implica que ∠PDC = ∠PCD =90− 15 = 75 y luego ∠CPD = 30.

4. Sea ABC un triangulo rectangulo con ∠CAB = 90 (Ver figura 97). D es un punto sobrela prolongacion de BC tal que BD = BA. E es un punto en el mismo semiplano que Arespecto de BC, tal que CE ⊥ BC y ademas CE = CA. Mostrar que A,D y E estanalineados.

Figura 97

Solucion: Sea ∠CBA = 2θ; el 4ABD es isosceles y ∠BAD + ∠BDA = 2θ, por loque ∠BAD = ∠BDA = θ. Como ∠CAB = 90 entonces ∠ACB + ∠ABC = 90 y comoCE es perpendicular a BC entonces ∠ECA + ∠ACB = 90; por lo tanto, ∠ABC =∠ECA = 2θ. Con esto, como 4ECA es isosceles, ∠CEA = ∠CAE = 90 − θ. Luego,∠EAC + ∠CAB + ∠BAD = 180 y ası E, A y D estan alineados.

5. Dado un cuadrado ABCD, se construyen los triangulos equilateros ABP (exteriormente)y ADQ (interiormente). Probar que C, P y Q estan alineados. Figura 98.

73

Page 74: Geometria Plana

Solucion: Observe que ∠PAQ = ∠BAD = 90 y PA = BA = DA = DQ, por loque 4PAQ es triangulo rectangulo isosceles, y por tanto, ∠PQA = 45. Por otra parte,∠QDC = 90 − ∠ADQ = 30 y QD = AD = CD, es decir, el 4CDQ es isoscelescon el angulo comprendido entre lados iguales de 30, por lo que ∠DQC = 75. Ası,∠PQA+ ∠AQD + ∠DQC = 180 y por lo tanto, C, P y Q estan alineados.

Figura 98

6. En la figura 99, AB = BC = CD = DE = EF = FG = GA. Calcule la medida del∠DAE. Referenciasfig20

Figura 99

Solucion: Sea ∠DAE = θ. Como los triangulos ABC y AGF son isosceles, ∠ACB =∠AFG = θ. Calculando los angulos externos de 4ABC y 4AFG se tiene ∠FBC =∠CGF = 2θ. Como 4GFE y 4BCD son isosceles, ∠GEF = ∠BDC = 2θ. Calculandoangulos externos de 4ADC y 4AEF se obtiene ∠ECD = ∠DFE = 3θ. Como 4CDEy4FED son isosceles, ∠CED = ∠FDE = 3θ. Entonces, la suma de los angulos internosdel 4AED da θ + 3θ + 3θ = 180, de donde θ = 180

7.

7. (XXVIII Olimpiada Brasilena de Matematica) En la figura 100, AB = AC, AM = ANy ∠CAM = 30°, encuentre el valor del ∠BMN .

Solucion: Como4ABC y4AMN son isosceles, sean ∠ABC = ∠ACB = α y ∠AMN =∠ANM . Por la formula del angulo externo se tiene

74

Page 75: Geometria Plana

Figura 100

∠ACM + ∠MAC = ∠AMB = ∠AMN + ∠BMNα + 30 = ∠ANM + ∠BMN

= (∠NBM + ∠BMN) + ∠BMN= α + 2∠BMN

Esto implica que ∠BMN = 15°.

8. (Etapa semifinal Estatal de XXII Olimpiada Mexicana de Matematicas) En la figura101 se muestra un hexagono regular ABCDEF de lado 1. Los arcos del cırculo que estandibujados tienen centro en cada vertice del hexagono y radio igual a la distancia al verticeopuesto. P , Q, R, S, T y U son los puntos de corte de estos arcos. ¿Cuanto mide cadalado del hexagono PQRSTU?

Figura 101

Solucion: El hexagono PQRSTU es regular y con el mismo centro que ABCDEF . SeaO el centro de ambos (Vease Figura 64). El lado buscado es igual a OP . Tenemos queCF = FP = PC = 2 por ser radios de los arcos dibujados; entonces CFP es equilaterode lado 2 y OP es una altura de este triangulo que, por Pitagoras, es igual a

√3.

75

Page 76: Geometria Plana

Congruencia de Triangulos

1. En la figura 102, ABC es un triangulo equilatero y CDEF es un cuadrado. Se construyeun punto G tal que CF = CG y ademas ∠CFG = 15°. Probar que ∠AGC = ∠BDC.

Figura 102

Solucion: ∠BCD = 180− ∠ACB − ∠DCF = 30. Como 4GCF es isosceles, ∠CGF =∠CFG = 15 y ∠ACG = ∠CGF + ∠CFG = 30. Por criterio LAL, 4BCD ≡ 4ACG,por lo tanto ∠BDC = ∠AGC.

2. (Cuaderno de Olimpiadas Mexicanas - Geometrıa) En la figura 103, ABCD un cuadradoy EF ⊥ GH. Demuestre que que EF = GH.

Figura 103

Solucion: Se construyen EK y GM con K sobre CD y M sobre AD tales que EK ‖ ADy GM ‖ CD. Luego se demuestra que 4EFK ≡ 4GHM , con EF = GH.

3. (Examen final de XVI Olimpiada mexicana de Matematica) Los angulos de un trianguloABC estan en progresion aritmetica (∠B − ∠A = ∠C − ∠B = θ), D, E, y F son lospuntos medios de los lados BC, CA y AB, respectivamente. Llamamos H al pie de laaltura trazada desde C (que cae entre B y F ) y G a la interseccion entre DH y EF .¿Cuanto vale ∠FGH?

Solucion: Note que ∠A + ∠B + ∠C = 3∠A + 3θ = 180, lo cual implica que ∠A + θ =

76

Page 77: Geometria Plana

60 = ∠B. Entonces 4BCH es un triangulo 30, 60, 90, y dado que D es punto medio deBC, el 4BDH es equilatero. Luego, como BC ‖ EG, ∠FGH = ∠BDH = 60. Ver figura104.

Figura 104

4. Sea ABCD un cuadrado. Se construyen triangulos equilateros ADP y ABQ como semuestra en la figura 105. Sea M la interseccion de CQ con AD y N la interseccion deCP con AB. Demuestre que CMN es un triangulo equilatero.

Figura 105

Solucion: Note que PD = AD, porque 4APD es equilatero, y AD = CD porqueABCD es cuadrado, por lo que PD = CD, es decir, el 4CDP es isosceles, con ∠CDP =∠CDA−∠PDA = 30, entonces ∠DPC = ∠DCP = 75, y ∠BCN = ∠BCD−∠DCP =15. Analogamente, 4BCQ es isosceles con angulos 30, 75, 75, por lo que ∠MCN =∠BCQ − ∠BCN = 60. Finalmente, como la figura es simetrica con respecto a AC,CM = CN , entonces, el triangulo CMN es equilatero porque tiene dos lados iguales yun angulo interno igual a 60.

5. 4ABC es un triangulo isosceles con ∠ABC = ∠ACB = 80°. D es un punto en AC talque ∠ABD = 10°. Demuestre que AD = BC. Figura 106.

77

Page 78: Geometria Plana

Solucion: Se traza un punto D′ sobre AC tal que AD′ = BC. Se construye exteriormenteel triangulo equilatero AEB. Luego, AE = AB, D′A = CB y ∠EAD′ = ∠ABC locual implica que 4EAD′ ≡ 4ABC, de donde se deduce que el 4D′EB es isoscelesy ∠BED′ = ∠BEA − ∠D′EA = 40. Se sigue que ∠EBD′ = 70 y como ∠D′BA =∠EBD′ − ∠ABE = 10, resulta que D′ = D y por lo tanto BC = AD′ = AD.

Figura 106

78

Page 79: Geometria Plana

Cuadrilateros.

1. Sea ABCD un paralelogramo. Se construyen triangulos equilateros exteriores 4CDP y4ADQ, como se muestra en la figura 107. Demuestre que el 4BPQ es equilatero.

Figura 107

Solucion: Observe que al hacer una rotacion de centro P y angulo 60, el trianguloPBC se transforma en el triangulo PQD (observe los segmentos PC y CB tras estatransformacion), mientras que al hacer una rotacion de centro Q y angulo 60, el trianguloPQD se transforma en triangulo BQA. Como la rotacion mantiene las distancias, PB =PQ = BQ, por lo que el trıangulo BPQ es equilatero.33

2. (II Olimpiada Matematica del Cono Sur) En la figura 108 ABCD y AECF son parale-logramos. Demuestre que BEDF es paralelogramo.

Figura 108

33Una demostracion mas rigurosa se basa en el calculo de los angulos ∠PCB = ∠PDQ = ∠BAQ = 120 +∠ABC y en la utilizacion del criterio LAL para justificar 4PCB ≡ 4PDQ ≡ 4BAQ.

79

Page 80: Geometria Plana

Solucion 1: Sea M el punto medio de AC. Las diagonales AC y BD se bisecan en M ,mientras que las diagonales AC y EF tambien se bisecan en M , entonces BD y EF sebisecan en M por lo que BEDF es un paralelogramo.Solucion 2: Como AD ‖ CB y AE ‖ CF entonces ∠DAE = ∠BCF . Entonces, porpropiedades de paralelogramos ∠BAE = ∠BAD−∠EAD = ∠BCD−∠BCF = ∠FCD;ademas, AB = CD y AE = CF . Por criterio LAL, 4BAE ≡ 4DCF , y entonces BE =DF . Analogamente se demuestre que 4ABF ≡ 4CDE, lo cual implica BF = DE.Como BEDF es un cuadrilatero con lados opuestos iguales, es un paralelogramo.

3. ABCD es un cuadrilatero convexo y O es un punto en su interior. Sean P , Q, R, S,los puntos medios de los lados AB, BC, CD, DA, respectivamente. Por P se traza unaparalela a OR, por Q se traza una paralela a OS, por R se traza una paralela a OP , ypor S se traza una paralela a OQ. Demuestre que estas cuatro rectas concurren.

Solucion: Al tomar las rectasOP yOR y sus paralelas se forma el paralelogramo PORM ,y al tomar las rectas OQ y OS y sus paralelas se forma el paralelogramo OQNS. Por elteorema de Varignon, sabemos que PQRS es un paralelogramo, y llamaremos T al puntode corte de sus diagonales. Observe que el punto de corte de las diagonales de PORM esel punto medio de PR, i.e., T ; analogamente, el punto de corte de las digonales de OQNSes el punto medio de SQ, i.e., T nuevamente. Ası, M es la reflexion de O con respecto aT , y de igual forma queda definido N , por lo que M = N y las cuatro rectas concurren.

4. (Hector Alberti) Sea ABCD un cuadrado. Se construyen los triangulos equilateros BDA′,ACB′, BDC ′ y ACD′ (Vease figura 109). Demuestre que el A′B′C ′D′ es tambien uncuadrado.

Figura 109

Solucion: Como A′B = A′D, AB = AD, CB = CD, C ′B = C ′D, los puntos A′, A,C, C ′ pertenecen a la mediatriz de BD, y por tanto, estan alineados. Analogamente, B′,B, D, D′ estan alineados; por lo tanto A′C ′ ⊥ B′D′. Por otra parte, si O es el centrode ABCD, como los triangulos equilateros construidos son todos iguales (tienen ladosiguales a la diagonal de ABCD) de altura h, OA′ = OB′ = OC ′ = OD′ = h. Entonces,A′B′C ′D′ es un cuadrilatero con diagones que se bisecan en O (es paralelogramo), soniguales A′C ′ = B′D′ = 2h (es rectangulo) y son perpendiculares (es rombo), lo cualimplica que es cuadrado.

80

Page 81: Geometria Plana

5. Un trapecio isosceles tiene diagonales perpendiculares y su area es 2010, determine sualtura.

Solucion: Considere la figura 110. Sea ABCD el trapecio del problema (AB = CD),como es trapecio isosceles, es simetrico con respecto a la mediatriz de las bases, en parti-cular, AC = BD. Sean P y Q los pies de las perpendiculares a AD trazadas desde B yC, respectivamente. Por LAL, 4ABD ≡ 4DCA lo cual implica ∠CAD = ∠BDA = 45(debido a que AC ⊥ BD) luego en el triangulo rectangulo 4ACQ, ∠ACQ = 45 por loque AQ = CQ y es facil ver que BC = PQ. Luego

2010 = (ABCD) =(CQ)(BC + AD)

2

=(CQ)(PQ+ AP + PQ+QD)

2= (CQ)(AP + PQ)

= CQ2

Luego CQ =√

2010.

Figura 110

6. (IX Competencia de Clubes Cabri, Segunda Ronda) Sea ABCDEF un hexagono regularcuyo centro es O. Se construyen los cuadrados FSOP y ORCQ. Demuestre que APQBy SEDR son rectangulos. Figura 111.

Solucion: Por construccion PF = PO = SF = SO, y por propiedades de hexagonoregular34 AF = AO = EF = EO, entonces P , S A, E, pertenecen a la mediatriz de FOy por tanto, estan alineados sobre una recta perpendicular a FO. Analogamente, Q, R,B, D estan alineados sobre una recta perpendicular a CO; ademas, es facil demostrarque AP = BQ = DR = ES. Observe ademas que AB ‖ CF ‖ DE, lo cual implica(AB ‖ CF ) ⊥ (AP ‖ BQ), es decir, APQB es rectangulo, y analogamente para SEDR.

34Los triangulos OAB, OBC, OCD, ODE, OEF , OFA son equilateros.

81

Page 82: Geometria Plana

Figura 111

7. Sobre los lados del 4ABC se trazan exteriormente los cuadrados ABPQ, CARS yBCTU . Luego se trazan los paralelogramos AQA′R, CSC ′T y BUB′P , como en la figura112.

a) Sean A′′, B′′, C ′′ los centros de los cuadrados BCTU , CARS, ABPQ, respectiva-mente. Demuestre que estos centros estan sobre los lados del 4A′B′C ′.

b) Demuestre que AA′′, BB′′, CC ′′ concurren.

Solucion:

a) Observe que al hacer una rotacion de centro A′′ y angulo igual a 90, el 4A′′UB′ setransforma en el 4A′′BA, y a la vez este ultimo se transforma en el 4A′′CC ′ (esto esporque A′′U 7→ A′′B 7→ A′′C y UB′ 7→ BA 7→ CC ′); esto significa que A′′B′ ⊥ A′′Ay A′′A ⊥ A′′C ′, por lo que B′, A′′, C ′ estan alineados, es decir, A′′ pertenece a B′C ′.Analogamente se prueban los otros casos.

b) De lo anterior, observe que AA′′ es mediatriz de B′C ′, por lo que AA′′, BB′′, CC ′′

concurren en el circuncentro del 4A′B′C ′.

8. Se dibujan cuadrados exteriores a los lados de un paralelogramo (Vea figura 113), de-muestre que:

a) El cuadrilatero determinado por los centros de esos cuadrados es un cuadrado.

b) Las diagonales de ese cuadrado son concurrentes con las del paralelogramo.

Solucion:

82

Page 83: Geometria Plana

Figura 112

a) Observe que al hacer una rotacion de centro O2 y angulo igual a 90, el 4O2BO1 setransforma en el 4O2CO3 (observe que los segmentos O2B y BO1 se transformanen O2C y CO3, respectivamente), por lo que O2O1 = O2O3 y O2O1 ⊥ O2O3. Re-pitiendo este razonamiento, O1O2 = O2O3 = O3O4 = O4O1 y estos segmentos sonperpendiculares si son consecutivos, por lo que O1O2O3O4 es un cuadrado.

b) Basta demostrar que AC y O1O3 se bisecan,35 y esto es equivalente a demostrarque AO1CO3 es un paralelogramo. Esto es cierto porque AO1 = CO3 y AO1 ‖ CO3

(ambos segmentos son perpendiculares a O1B)

Figura 113

35Porque ası los puntos de corte de las diagonales de ABCD y O1O2O3O4 coincidirıan.

83

Page 84: Geometria Plana

9. Dado un 4ABC, se construyen exteriormente los triangulos rectangulo isosceles 4ACPy 4BCQ, con AC y BC como hipotenusas. Si M es el punto medio de AB, demuestreque el 4MPQ tambien es un triangulo rectangulo isosceles.

Solucion: Construya los cuadrados exteriores ACDE y BCFG, como muestra la figura114. Observe que P y Q son los puntos medios de AD y BF , respectivamente. Al rotarel 4BCD con centro C y angulo de 90, se genera el 4FCA, entonces dichos triangulosson congruentes y en por tanto BD = AF y BD ⊥ AF . Por otra parte, observe que MPes base media del 4BAD, por lo que 2MP = BD y MP ‖ BD; analogamente, MQ esbase media del 4ABF , por lo que 2MQ = AF y MQ ‖ AF . Por lo tanto MP = MQ yMP ⊥MQ.

Figura 114

84

Page 85: Geometria Plana

Angulos en Circunferencia.

1. Dada la figura 115, demuestre que AB ‖ A′B′.

Figura 115

Solucion: Observe que los cuadrilateros ABQP y A′B′QP son cıclicos, por lo que∠PAB = ∠PQB′ = 180°−∠PA′B′, por lo tanto AB ‖ A′B′.

2. Dos circunferencias de centros O1 y O2 son tangentes (interna o externamente) en unpunto P ; por este punto se traza una recta que corta nuevamente a la circunferencias enA y B, respectivamente. Demuestre que AO1 ‖ BO2.

Solucion: En la figura 116 se ha considerado que las circunferencias son tangentes exte-riormente, sin embargo, el otro caso es analogo. Se sabe que O1, P , O2 estan alineados, yque 4APO1 y BPO2 son triangulos isosceles (dos de sus lados son radios de una circun-ferencia), entonces ∠O1AP = ∠APO1 = ∠BPO2 = ∠O2BP , por lo que AO1 ‖ BO2.

Figura 116

3. Dadas dos circunferencias una fuera de la otra como en la figura 117, demuestre que lastangentes comunes externas forman segmentos iguales; analogamente, las tangentes co-munes internas forman segmentos iguales.

Solucion:36 Sea P la interseccion de las tangentes comunes externas AA′ y BB′. Entonces

36Suponemos que las circunferencias tienen radios distintos; cuando los radios son iguales, el problema sejustifica por la simetrıa de la figura.

85

Page 86: Geometria Plana

AA′ = PA′−PA = PB′−PB = BB′. Analogamente se resuelve el caso de las tangentescomunes internas.

Figura 117

4. Teorema de Pithot. Demuestre que en todo cuadrilatero inscribible, la suma de ladosopuestos es igual.37

Solucion: Considere la figura 118, ABCD es el cuadrilatero inscribible, con P , Q, R, S,los puntos de tangencia sobre AB, BC, CD, DA, respectivamente. Entonces

AB + CD = AP + PB + CR +RD

= AS +BQ+ CQ+DS

= BC +DA

Figura 118: Teorema de Pithot.

5. Teorema de Steiner. En todo cuadrilatero exinscrito a una circunferencia, la diferenciade las longitudes de lados opuestos es igual.

Solucion: El cuadrilatero puede quedar en posiciones como las de la figura ??; en amboscasos, la demostracion es muy similar, y analoga a la de Pithot. Para la figura de la

37El recıproco de este teorema y del siguiente son tambien es ciertos.

86

Page 87: Geometria Plana

izquierda se tiene que

AB − CD = (AP −BP )− (CR−RD)

= (AS −BQ)− (CQ−DS)

= AD −BC

Figura 119: Teorema de Steiner.

6. Teorema de Miquel: Dado un 4ABC, sean X, Y , Z puntos sobre AB, BC, CA,respectivamente . Demuestre que los circuncırculos de 4AXZ, 4BYX, 4CZY tienenun punto en comun M .

Solucion: Sea M el otro punto de corte de los circuncırculos de 4AXZ y 4BYX. Comolos cuadrilateros AXMZ y BYMX son cıclicos, se tiene

∠YMZ = 360− ∠XMZ − ∠YMX

= 360− (180− ∠A)− (180− ∠B)

= 180− ∠C

Entonces, CYMZ es cuadrilatero cıclico, por lo que M esta sobre el circuncırculo del4CZY .

Figura 120: Teorema de Miquel.

87

Page 88: Geometria Plana

7. Sea ABC un triangulo, y sean L y N las intersecciones de la bisectriz del angulo A conel lado BC y el circuncırculo de ABC respectivamente (Ver figura 121). Construimos lainterseccion M del circuncırculo de ABL con el segmento AC. Prueba que los triangulosBMN y BMC tienen la misma area.

Solucion: Observe que ABNC y ABLM son cuadrilateros cıclicos, por lo que ∠NCB =∠NAB = ∠LAM = ∠LBM , por lo que CN ‖ BM . Entonces, las distancias de N y C ala recta BM son iguales, y por tanto, el area del 4BMN es igual al area del 4BMC.

Figura 121

8. Sea AB el diametro de una semicircunferencia. Se colocan los puntos M y K sobre lasemicircunferencia y sobre AB, respectivamente.38 Sea P el centro de la circunferenciaque pasa por A, K y M ; sea Q el centro de la circunferencia que pasa por B, K y M .Demuestre que MPKQ es concıclico.

Solucion: Como AB es diametro, ∠AMB = 90, entonces ∠MAB + ∠MBA = 90. Ası

∠MPK + ∠MQK = 2∠MAK + 2∠MBK

2 = 2 (∠MAB + ∠MBA)

= 180

Por lo tanto, MPKQ es concıclico.

38M y K son distintos de A y B.

88

Page 89: Geometria Plana

9. Las circunferencias Γ1 y Γ2 se cortan en los puntos A y B. Por el punto A se traza una rectaque corta nuevamente a las circunferencias Γ1 y Γ2 en los puntos C y D, respectivamente.Por los puntos C y D se trazan tangentes a las circunferencias, las cuales se cortan en elpunto M . Demuestra que MCBD es cıclico. Figura 122.

Solucion: Es suficiente probar que MCBD es un cuadrilatero con un par de angulosopuestos suplementarios. Por angulos seminscritos y suma de angulos internos de untriangulo, se tiene

∠CMD + ∠CBD = ∠CMD + ∠CBA+ ∠DBA

= ∠CMD + ∠MCA+ ∠MDA

= 180

Figura 122

10. El 4ABC cumple que ∠A = 90° y AB = AC. Se toma un punto E del segmento AB, seconstruye interiormente un triangulo equilatero AEF . EF corta BC en I, y se construyeexteriormente un triangulo equilatero BIJ . Encuentre ∠EJB.

Solucion: Como el ∠BJI = 60◦ = ∠AEI,el cuadrilatero BEIJ es cıclico, por lo que el∠EJB = ∠EIB = ∠AEI − ∠EBI = 15°.

89

Page 90: Geometria Plana

11. En la figura 123, se sabe que ∠AO1B − ∠AO2B = 70◦ y ademas la tangente EB formael triangulo isosceles ABE, con AB = AE. Encuentre ∠EBC.

Figura 123

Solucion: Sea ∠AO2B = 2θ, entonces ∠ACB = θ y por hipoteis ∠AO1B = 2θ+ 70. Porangulo seminscrito, ∠ABE = θ + 35, y como el 4ABE es isosceles, ∠AEB = θ + 35.Finalmente, por la formula del angulo externo aplicada al 4BCE, ∠EBC = ∠AEB −∠ECB = 35

12. Dos circunferencias Γ1 y Γ2 se cortan en A y B. Una recta por A corta a Γ1 y Γ2 en C yD, respectivamente, y la paralela a CD por B corta Γ1 y Γ2 en E y F , respectivamente.(Ver figura 124). Demuestre que 4CDB ≡ 4EAF .

Solucion: Sean G = AE ∩ BC y H = AF ∩ BD. Como AC ‖ BE y ACEB es cıcli-co,39 ∠CAG = ∠GEB = ∠ACG = ∠GBE = α; analogamente, ∠DAH = ∠HFB =∠ADH = ∠FBH = β. Observe que 4GAC y 4GBE son triangulos isosceles y por tan-to AE = AG+GE = CG+GB = CB; de forma similar se obtiene AF = DB. Finalmente,∠EAF = 180− α− β = ∠CBD, por lo que, por el criterio LAL, 4CBD ≡ 4EAF .

13. La Recta de Simson-Wallace. Sean X, Y y Z los pies de las alturas trazadas desde unpunto P en el circuncırculo del 4ABC hacia AB, BC y CA, respectivamente. Demuestreque X, Y y Z estan alineados.40

Solucion: Como BPY X es cıclico, ∠Y XP = Y BP = θ. Como ABPC es cıclico,∠CBP = ∠CAP = θ. Como AXPZ es cıclico, ∠ZAP = ∠ZXP = θ. Por lo tanto,dado que ∠Y XP = ZXP , los puntos X, Y y Z estan alineados.

39ACEB es un trapecio isosceles.40El recıproco tambien es cierto, si X, Y y Z estan alineados, entonces P debe estar sobre el circuncırculo

del 4ABC; en cualquier otro caso, el 4XY Z se llama el triangulo pedal con respecto al punto P .

90

Page 91: Geometria Plana

Figura 124

Figura 125: Recta de Simson-Wallace

14. Sea P un punto exterior al cuadrado ABCD tal que ∠APC = 90◦, Q es la interseccionde AB y PC, y R el pie de la perpendicular por Q a CA. Demuestre que P , R y D estanalineados.

Solucion: Como ∠APC + ∠ADC = 180°, el cuadrilatero PADC es cıclico, entonces∠APD = ∠ACD = 45°. Analogamente, como ∠APQ + ∠ARQ = 180°, el cuadrilateroPARQ es cıclico, entonces ∠APR = ∠AQR = 90°−∠QAR = 45°. Por lo tanto, como∠APD = ∠APR, los puntos P , R, D estan alineados. Figura 126.

15. (OIM 2002, P-4) En un triangulo escaleno ABC se traza la bisectriz interior BD, con Dsobre AC. Sean E y F puntos sobre la recta BD tales que (AE ‖ CF ) ⊥ BD, y sea M elpunto sobre el lado BC tal que DM ⊥ BC. Demuestre que ∠EMD = ∠DMF . Figura127.

91

Page 92: Geometria Plana

Figura 126

Solucion: Como DM ⊥ MC y DF ⊥ FC, DFCM es cıclico, por lo tanto ∠DMF =∠DCF = θ, y como AE ‖ FC, entonces ∠EAD = ∠DCF = θ. Sea G la interseccionde AE con BC. Como AG ⊥ BE, BE es altura y bisectriz del 4ABG, por lo que estetriangulo es isosceles y ademas BE es mediatriz de AG; entonces ∠EGD = ∠EAD = θ.Y finalmente, podemos ver que DEMG es cıclico, pues ∠DEG = ∠DMG = 90◦, ası que∠EMD = ∠EGD = θ. De aquı, el resultado es inmediato.

Figura 127

16. (OMCC 2003, P-2) Sea S una circunferencia y AB un diametro de ella. Sea t la rectatangente a S en B y considere dos puntos C y D en t tales que B este entre C y D. SeanE y F las intersecciones de S con AC y AD y sean G y H las intersecciones de S conCF y DE. Demuestre que AH = AG.

Solucion: Como AEBF es cıclico (Ver figura 128), ∠AEF = ABF . Luego, como AB ⊥CD y BF ⊥ AD, se cumple tambien ∠ABF = ∠FDB, por lo que ∠AEF = ∠FDC, esdecir, el cuadrilatero CDFE es cıclico. Utilizando este resultado y el hecho que EGHFtambien es cıclico, se tiene ∠EDC = ∠EFG = ∠EHG, por lo que CD ‖ GH. Esto

92

Page 93: Geometria Plana

implica que AB ⊥ GH, y como AB pasa por el centro de la circunferencia, debe sermediatriz de GH, por lo tanto AG = AH.

Figura 128

17. (The 59th Romanian Mathematical Olympiad District Round) Considere un cuadradoABCD y un punto E sobre el lado AB. La diagonal AC corta al segmento DE en el pun-to P . La perpendicular por P a DE corta al lado BC en F . Probar que EF = AE+CF .

Solucion: Se construye E ′ sobre BC de tal manera que CE ′ = AE (como se mues-tra en la figura 129) y que C quede entre F y E ′, ası por LAL se tiene que los triangulosrectangulos 4DAE ≡ 4DCE ′ por lo tanto ∠ADE = ∠CDE ′ luego ∠EDE ′ = 90. Porotra parte, el cuadrilatero DCFP es cıclico, por lo que ∠PDF = ∠PCF = 45 entonces∠FDE ′ = ∠EDE ′ − ∠EDF = 45. Ahora por LAL los triangulos 4DEF ≡ 4DE ′F ,por lo que EF = E ′F = E ′C + CF = AE + CF .

Figura 129

93

Page 94: Geometria Plana

18. Teorema de Arquımedes: En la figura 130, la region delimitada por tres semicircunfe-rencias mutuamente tangentes, es conocida como cuchilla de zapatero o arbelos. Demostrarque las circunferencias sombreadas son congruentes.

Figura 130: Teorema de Arquımedes.

Solucion: Sean AB, AC, BC los diametros de las semicircunferencias que forman el arbe-los, de radios r, r1, r2 y centrosO,O1,O2, respectivamente. De momento nos concentramosen el lado izquiero de la figura Referenciasfigura58; sea C1 el centro de la circunferencia dela izquierda y R1 su radio; D, E y F son los puntos de tangencia de esta circunferencia condos semicircunferencias del arbelos y con la recta perpendicular a los diametros por C; fi-nalmente, G es la proyeccion de C1 sobre AB. En primer lugar, OO1 = OA−O1A = r−r1.Por otra parte, observe que O, C1, D estan alineados y O1, C1, E tambien estan alineados,entonces OC1 = OD − C1D = r − R1 y O1C1 = O1E + EC1 = r1 + R1. Ademas, comoCFC1G es un rectangulo, GC = FC1 = R1, entonces O1G = O1C − GC = r1 − R1 yOG = O1G−O1O = (r1 −R1)− (r − r1) = 2r1 − (r +R1). Ahora, aplicando el teoremade Pitagoras a 4GO1C1 y 4GOC1 se tiene

O1C21 −O1G

2 − C1G2 = OC2

1 −OG2 − C1G2

(r1 +R1)2 − (r1 −R1)

2 = (r −R1)2 − (2r1 − (r +R1))

2

4r1R1 = −4rR1 − 4r21 + 4r1r + 4r1R1

⇒ R1 =r1(r − r1)

r

Figura 131

Analogamente, si r2 y R2 son los radios del semicırculo y del cırculo de la derecha, res-pectivamente, entonces

R2 =r2(r − r2)

r

94

Page 95: Geometria Plana

Pero 2r = AB = AC + BC = 2r1 + 2r2, entonces r2 = r − r1, y sustituyendo en laecuacion anterior se tiene

R2 =(r − r1) (r − (r − r1))

r=

(r − r1)r1r

= R1

95

Page 96: Geometria Plana

Teorema de Thales y Semejanza.

1. (IV OMCC, P-4) Sea ABC un triangulo, D el punto medio de BC, E un punto sobreel segmento AC tal que BE = 2AD y F el punto de interseccion de AD con BE. Si∠CAD = 60°, encuentre la medida de los angulos del 4FEA. Figura 132.

Solucion: Se traza por D una paralela a BE y sea G el punto por el que esta paralelacorta al lado AC. Como DG es base media del 4BCE se tiene que DG = BE

2= AD;

entonces 4ADG es isosceles y tiene un agulo de 60, por lo que debe ser equilatero.Finalmente 4AEF tambien es equilatero, por tanto sus angulos son iguales a 60.

Figura 132

2. Sea ABCD es un trapecio con AD ‖ BC. M y N son los puntos medios de CD y BC,respectivamente, y P el punto comun de las rectas AM y DN . Si PM

AP= 1

4, demuestre que

ABCD es paralelogramo.

Solucion: Sea Q el punto medio de DN , entonces QM ‖ BC ‖ DA. Como MQ esbase media del 4CDN , MQ = CN

2= CB

4. Por otra parte, como 4PMQ ' 4PAD,

MQAD

= PMAP

= 14, entonces MQ = AD

4. Finalmente, como BC ‖ DA y BC = DA, ABCD

es paralelogramo.

3. En la figura 133, BC = CD = DE = EA = x y ∠AEB = 90°. Demuestre que ∠ABC +∠ACD + ∠ADE = 90°.

Figura 133

Solucion 1: Por Pitagoras, AD =√

2x. Observe que DA2 = 2x2 = DB ·DC, por lo que4ABD ' 4CAD; entonces ∠ABD = ∠CAD y por tanto ∠ABC + ∠ACD + ∠ADE =

96

Page 97: Geometria Plana

∠CAD + ∠ACD + ∠ADE = 2∠ADE = 90.

Solucion 2: Considere la siguiente cuadrıcula (Figura 134). Observe que al hacer unarotacion de centro A y angulo igual a 90, el segmento AC se transforma en AF , por loque el 4ACF es triangulo rectangulo isosceles, y ∠ADE = ∠ACF . Se cumple ∠ABC =∠CFD, porque se forman con la diagonal de tres cuadrados; analogamente, ∠ACD =∠AFD, porque se forman con la diagonal de dos cuadrados. Sumando los angulos internosdel 4ACF se obtiene el resultado buscado.

Figura 134

4. (Asiatico Pacıfica) Sea ABC un triangulo y D el pie de la altura con respecto a A. Sean Ey F puntos en una recta que pasa por D (distintos de D) tales que AE ⊥ CE y AF ⊥ BF .Sean M y N los puntos medios de BC y EF , respectivamente. Demuestre que AN ⊥ NM .

Solucion: En la figura 135, AE ⊥ CE y AD ⊥ DC entonces, ADEC es cıclico, ası que∠DEA = ∠DCA. Del mismo modo, como AF ⊥ BF y AD ⊥ DB, AFBD es cıclicoy entonces ∠AFD = ∠ABD. Esto implica que 4ABC ' 4AFE y a partir de estasemejanza, 4ABM ' 4AFN . Luego, ∠AMB = ∠ANF , por lo que el cuadrilateroANDM es cıclico, y por lo tanto ∠ANM = ∠ADM = 90.

97

Page 98: Geometria Plana

Figura 135

Puntos y rectas notables.

1. De acuerdo con los datos de la grafica 136, calcular el valor de AB.

Figura 136

Solucion 1: Por el teorema de la bisectrız ACAB

= CDDB

, de donde AC = 54x, luego, aplicando

el teorema de pitagoras al 4ABC, se tiene que x2 + 182 =(

54x)2

, que despues de resolverse tiene que x = 24.

Solucion 2: Dado que D es un punto de la bisectrız del ∠BAC, entonces D equidistade los lados de dicho angulo, sea pues H ∈ AC talque DH⊥AC y DH = DB = 8entonces, aplicando el teorema de pitagoras en el 4CDH se deduce que HC = 6, por loque AC = x+ 6, y por el teorema de pitagoras en el 4ABC, x2 + 182 = (x+ 6)2, por lotanto, x = 24.

2. Sea ABCD un paralelogramo. Q es el punto medio de AD,F el pie de la perpendicularpor B sobre QC. Probar que AF = AB.Solucion: Sea E el punto medio de BC y G la interseccion de AE con BF . ComoAE ‖ CQ, se tiene que AG ⊥ BF . Pero tambien, como AE ‖ CQ, entonces EG ‖ CFpor lo que en el 4BCF , EG es base media. Entonces BG = GF de donde se sigue que4ABF es isosceles porque BG es altura y mediana.

98

Page 99: Geometria Plana

3. Sea ABCD un cuadrilatero tal que AB = CD. Las mediatrices de AC y BD se cortanen P . Probar que ∠PAC = ∠PCA = ∠PBD = ∠PDB. Figura 137.Solucion: Como P esta sobre las mediatrices de AC y BD, PA = PC y PB = PD,y por hipotesis, AB = CD, entonces por criterio LLL, 4ABP ≡ 4CDP . De aquı,∠APB = ∠CPD, entonces ∠BPD = ∠APD + ∠APB = ∠APD + ∠CDP = ∠APC;por lo tanto, 4BPD ' 4CPA, dada la igualdad anterior y el hecho que son triangulosisosceles. De esta semejanza se obtiene ∠PAC = ∠PCA = ∠PBD = ∠PDB.

Figura 137

4. ABC es un triangulo y P un punto en su interior. Sean A′, B′ y C ′ las reflexiones deP sobre BC, CA y AB, respectivamente. D, E y F son los pies de las perpendicularesrespectivos desde A, B y C hacia B′C ′, C ′A′ y A′B′. Probar que AD, BE y CF sonconcurrentes. Figura 138.

Solucion: Por propiedades de reflexion axial AC ′ = AP = AB′, por lo que el 4AB′C ′ esisosceles, y entonces AD es mediatriz de B′C ′. Analogamente, BE es mediatriz de C ′A′,mientras que CF es mediatriz de A′B′. Por lo tanto, las rectas AD, BE, CF concurrenen el circuncentro del 4A′B′C ′.

5. (Arnoldo Aguilar) En la figura 139, ABGH, BCFG y CDEF son cuadrados. Si I es elcentro de ABGH y J = DH ∩BG, demuestre que I, J y F estan alineados.

Solucion: Como G es punto medio de HF , BG es una mediana del 4BFH. Ademas,BDFH es un paralelogramo, luego sus diagonales BF y DH se cortan en su punto medio,digamos K. Se sigue que HK es tambien una mediana del 4BFH, y en consecuencia elpunto de corte de J = KH ∩BG es el centroide del 4BFH. Pero I es el punto medio deBH, ası que FI es la tercera mediana del 4BFH, por lo tanto J esta sobre el segmentoFI.

99

Page 100: Geometria Plana

Figura 138

Figura 139

6. (Arnoldo Aguilar) Sea ABC un triangulo equilatero. M y N son los puntos medios de ABy BC, respectivamente. Exteriormente al 4ABC se construye un triangulo rectanguloisosceles 4APC, con ∠APC = 90◦. Si I es la interseccion de AN y MP , demuestre queCI es la bisectriz de ∠ACM .

Solucion: Observe que AN es bisectriz del ∠BAC.Como ∠APC = ∠BMC = 90, el cuadrilateroAPCM es cıclico, por lo que ∠PMC = ∠PAC =∠PCA = ∠PMA = 45, entonces MP es bisectrizdel ∠AMC. De aquı se concluye que I = MP ∩ ANes el incentro del 4ACM , por lo que CI es bisectrizdel ∠ACM .

7. En la figura 140, el 4ABC es tal que ∠A = 90 y ∠B = 60. ¿Cual es el radio de lacircunferencia?

Solucion: Por relaciones de triangulos notables, BC = 2 y CA =√

3. Sean P y Qlas proyecciones de O sobre AB y AC respectivamente; por construccion, APOQ es unrectangulo, pero como OP = r = OQ, es tambien cuadrado, por lo que AP = r. Observe

100

Page 101: Geometria Plana

Figura 140

que la circunferencia es el excırculo del 4ABC, por lo que AP = s y entonces

r = s

=1 + 2 +

√3

2

=3 +√

3

2

8. Dado el paralelogramo ABCD, sea M el punto medio de AB, y N la interseccion de CDcon la bisectriz interna del ∠ABC. Demuestre que MC ⊥ BN si y solo si AN es bisectrizdel ∠DAB.

Solucion:(⇒) Si suponemos que MC ⊥ BN entonces BN es mediatriz de MC, y como BM ‖ CNentonces ∠CBN = ∠MBN = ∠CNB = ∠MNB, esto implica que BC ‖ MN , y portanto N es punto medio de CD; ası, AMND es un rombo y AN es bisectriz del ∠DAM .(⇐) Si suponemos que AN es bisectriz del ∠DAB, es propiedad conocida que AN ⊥ BN ,por lo que M es el circuncentro del4ABN y por la relacion entre angulo central y anguloinscrito se tiene ∠AMN = 2∠ABN = ∠ABC, por lo tanto MN ‖ BC y BCNM es unrombo, de donde se obtiene MC ⊥ BN .

9. En el 4ABC, se sabe que los vertices B, C, el circuncentro O y el ortocentro H del4ABC estan todos sobre una misma circunferencia. Figura 141.

a) Calcule el valor de ∠A.

b) Demuestre que el incentro tambien pertenece al circuncırculo de BCOH.

Solucion:

a) Sea ∠BAC = α. Como O es el circuncentro del4ABC, tenemos que ∠BOC = 2α. Porotra parte, sabemos que al ser H ortocentro, se cumple que ∠BHC = 180◦−α. Ahora

101

Page 102: Geometria Plana

bien, la condicion de que B, C, H y O son concıclicos implica que ∠BOC = ∠BHC,de donde 2α = 180◦ − α, y por tanto α = 60◦.

Figura 141

b) Este problema se basa en el siguiente resultado: si I es el incentro del 4ABC entonces∠BIC = 90 + ∠A

2. Como en este caso ∠A = 60, entonces ∠BIC = 120 = ∠BOC =

∠BHC, por lo que B, C, O, H, I, se ubican sobre una misma circunferencia.

10. Sea ABC un triangulo tal que las medianas respectivas a B y C son perpendiculares.Demuestre que se cumple la relacion (Ver figura 142).

5BC2 = CA2 + AB2.

Figura 142

Solucion: Sean BB′ y CC ′ las medianas que son perpendiculares, y sea G el centroide.Observe que el cuadrilatero BCB′C ′ tiene diagonales perpendiculares; por el teorema dePitagoras se cumple

BC2 +B′C ′2 = C ′B2 +B′C2

BC2 +

(BC

2

)2

=

(AB

2

)2

+

(AC

2

)2

5BC2 = AB2 + AC2

102

Page 103: Geometria Plana

11. Sea ABC un triangulo de ortocentro H. Sean P y Q los pies de las perpendiculares desdeH a las bisectrices interior y exterior de A, respectivamente. Si M es el punto medio deBC, mostrar que P , Q y M estan alineados. Figura 143.

Solucion: Sean E y F los pies de las alturas trazadas desde a B y C, respectivamente.Se sabe que AP ⊥ AQ, por lo que APHQ es un rectangulo. Como ∠APH = ∠AQH =∠AEH = ∠AFH = 90, los puntos P , Q, E, F , pertenecen a una circunferencia dediametro AH. Ademas, en esta circunferencia, como AP y AQ son bisectrices (interiory exterior, respectivamente) del ∠EAF , P y Q son los puntos medios del arcos EF ,por lo que PQ es la mediatriz de EF . Por otra parte, como ∠BEC = ∠BFC = 90, elcuadrilatero BCEF es cıclico, y el circuncentro es M , por lo que ME = MF ; entones Mesta en la mediatriz de EF , la cual es PQ.

Figura 143

12. En un triangulo ABC, sea M el punto medio de BC. Si se cumple que AB 6= AC yademas ∠MAC + ∠ABC = 90◦, hallar ∠BAC. Figura 144.

Solucion: Sin perdida de generalidad, suponga que AB > AC. Sea N la interseccion deAB con la mediatriz de BC. Se forma el 4BCN que es isosceles, entonces ∠CNM =90 − ∠MCN = 90 − ∠MBN = ∠CAN , lo cual implica que el cuadrilatero ACMN escıclico. Por lo tanto, ∠BAC = ∠BMN = 90.

13. Sea ABC un triangulo y U un punto de su circuncırculo tal que AU es bisectriz. Lasmediatrices en AB y AC cortan a AU en X y Y . Sea T la interseccion de BX con CY .Demostrar que AU = TB + TC. Figura 145.Solucion:41 Como X y Y pertenecen a las mediatrices de AB y AC, respectivamente,y a la bisectriz AU , entonces 4ABX y 4ACY cumplen ser isosceles y semejantes entresi, porque ∠XBA = ∠XAB = ∠Y AC = ∠Y CA = α. Esto implica ∠TXY = ∠XBA +

41El caso cuando AB = AC es trivial, porque X, Y y T colapsan en el circuncentro del 4ABC.

103

Page 104: Geometria Plana

Figura 144

∠XAB = 2α = ∠Y AC+∠Y CA = ∠TY X, es decir, el4TXY es isosceles con TX = TY .Por otra parte, como ABUC es cıclico, ∠UBC = ∠UAC = ∠UAB = ∠UCB = α. Deaquı se concluye que 4UBC es isosceles, con UB = UC. Ademas, ∠XUB = ∠ACB =∠Y CU y ∠XBU = ∠ABC = Y UC; por criterio ALA, 4UXB ≡ 4CY U , por lo queBX = Y U . Finalmente, TB + TC = (BX − TX) + (CY + TY ) = Y U + AY = AU .

Figura 145

14. (The 59th Romanian Mathematical Olympiad Final Round) Sea ABCD un rectangulo decentro O con AB 6= BC. La perpendicular en O a BD corta a las lıneas AB y BC en lospuntos E y F , respectivamente. Sean M y N los puntos medios de los segmentos CD yDA, respectivamente. Probar que las lıneas rectas FM ⊥ EN .

Solucion: Considere la figura 146, sin perdida de generalidad, se ha supuesto AB < BC.42

Sea L el punto medio de AB, y H es la interseccion de EF con AD. Se tiene queLN ‖ BD, y como BD ⊥ EF entonces LN ⊥ EF ; ademas, como ABCD es un rectangu-lo, DA ⊥ AB, por lo tanto, H es el ortocentro del 4ELN , y ası, LH ⊥ EN . Por otraparte, las reflexiones de L y H con respecto a O son respectivamente M y F , por lo queLH ‖MF , lo cual implica que FM ⊥ EN .

42El otro caso es completamente analogo.

104

Page 105: Geometria Plana

Figura 146

15. Sea ABC un triangulo rectangulo, con A = 90◦. Sea D un punto en su interior tal que∠DAC = ∠DCA = ∠DBC = α, y AC = BD. Determine el valor de α. Figura 147.

Solucion: Sean P y Q los pies de las perpendiculares trazadas desde D hacia CA y AB,respectivamente, R es un punto sobre BC tal que DB ⊥ DR, y E es la interseccion de CDcon AB. Como el 4ACD es isosceles, P es punto medio de AC, entonces AC = 2PA =2DQ = BD, por lo que el 4BDQ es un triangulo notable y ∠DBQ = 30. Por otra parte,por criterio ALA, 4ACE ≡ 4DBR, por lo que CE = BR; como PD ‖ AE, D es puntomedio de CE; ası, si M es el punto medio de BR (y circuncentro del 4BDR) se cumpleque DC = RM = DM , por lo que el 4CDM es isosceles. Por la relacion entre el anguloinscrito y el angulo central ∠DMR = 2∠DBR, por lo tanto ∠DCR = 2α. Sumando losangulos internos del 4ABC se tiene ∠A + ∠B + ∠C = 90 + 30 + α + 3α = 180, lo cualimplica α = 15.

Figura 147

16. Sea ABC un triangulo y M un punto tal que ∠MAB = 10, ∠MBA = 20, ∠MAC = 40

105

Page 106: Geometria Plana

y ∠MCA = 30. Probar que el 4ABC es isosceles. Figura 148.Solucion: Sea D la reflexion del punto A con respecto a la recta BM . Entonces el4AMDes isosceles con ∠AMD = 2 (∠MAB + ∠ABM) = 60 y por lo tanto es equilatero. Tam-bien ∠DBA = 2∠MBA = 40 y como ∠BAC = 50, implica que DB ⊥ AC. Sea E lainterseccion de BD con CM , se cumple que ∠CED = 90 − ∠ACE = 60 = ∠MAD,por lo que el cuadrilatero AMED es cıclico. De aquı, ∠DEA = ∠DMA = 60. Como∠DEC = ∠DEA y ED ⊥ AC, se tiene que ED es bisectriz y altura en el 4AEC, porlo tanto ED es mediatriz de AC, lo cual implica que BA = BC.

Figura 148

17. Teorema de Poncelet: Demuestre si 4ABC es un triangulo rectangulo con ∠A = 90°,entonces 2(r +R) = b+ c.

Figura 149: Teorema de Poncelet

Solucion: Sean O e I el circuncentro y el incentro del 4ABC. Como ∠A = 90°, O es elpunto medio de BC, por lo que a = 2R. Por otra parte, si P y Q son las proyecciones de Isobre AB y AC, claramente APIQ es rectangulo, pero como I es incentro IP = r = IQ,por lo que APIQ es cuadrado. Se sabe que para un triangulo cualquiera AP = s− a, porlo tanto

r = s− a

r =b+ c− a

22r + a = b+ c

2(r +R) = b+ c

106

Page 107: Geometria Plana

18. En la figura 150, ABCD y PQRS son cuadrados,4ABP ≡ 4BCQ ≡ 4CDR ≡ 4DASy los los radios de las cinco circunferencias son iguales a r. Si a es el lado del cuadradoABCD, determine r en funcion de a.

Figura 150

Solucion: Se tiene AB = a y se definen b = AP y c = BP ; observe que por las congruen-cias BQ = b, por lo que PQ = c−b = 2r. Por otra parte (analogamente a la demostraciondel teorema de Poncelet), al calcular el inradio del 4ABP se tiene que 2r = b + c − a,entonces c − b = b + c − a, lo cual implica que a = 2b. Por lo tanto, el 4ABP es untriangulo notable de 30, 60, 90, y ası

r =c− b

2

=

√3

2a− 1

2a

2

=

(√3− 1

4

)a

107

Page 108: Geometria Plana

19. Recta de Euler. El centroide G, el ortocentro H y el circuncentro O de un trianguloestan alineados, y ademas GH = 2GO.

Figura 151: Recta de Euler

Solucion: Considere la siguiente figura. Sean AHa y BHb alturas, OA′ y OB′ mediatrices.Observe que HA ⊥ BC y OA′ ⊥ BC, por lo que HA ‖ OA′; analogamente HB ‖ OB′;tambien, por el teorema de la base media AB ‖ A′B′ y AB = 2A′B′. Esto implica que4ABH ' A′B′O y la razon de semejanza es 2; en particular AH = 2A′O. Si definimosG como la interseccion de la mediana AA′ con HO, claramente 4AHG ' 4A′OG y larazon de semejanza es la misma que la anterior, por lo que GA = 2GA′, i.e., G es elcentroide del 4ABC. Esto implica que el ortocentro, el centroide y el circuncentro de untriangulo estan alineados, y por la semejanza GH = 2GO.

108